SóProvas



Questões de Probabilidade condicional, Teorema de Bayes e independência


ID
70744
Banca
FCC
Órgão
TRT - 3ª Região (MG)
Ano
2009
Provas
Disciplina
Estatística
Assuntos

Determinados processos de um tribunal são encaminhados para a análise de 3 analistas: X, Y e Z. Sabe-se que 30% de todos esses processos são encaminhados para X, 45% para Y e 25% para Z. Usualmente, por falta de documentação, uma parcela de tais processos é devolvida. Sabe-se que 5% , 10% e 10% dos processos de X, Y e Z, respectivamente, são devolvidos. A probabilidade de que um processo escolhido ao acaso tenha sido encaminhado para X, sabendo que foi devolvido, é

Alternativas
Comentários
  • Analistas Proc Encaminhados Devolvidos Não DevolvidosX 0,30 0,05 * 0,30 = 0,015 0,95 * 0,30 = 0,285Y 0,45 0,10 * 0,45 = 0,045 0,90 * 0,45 = 0,405Z 0,25 0,10 * 0,25 = 0,025 0,90 * 0,25 = 0,225Proporção do total 0,085 0,915 Após detalharmos as proporções de processos devolvidos e não devolvidos percebemos que os processos devolvidos por X equivalem a 1,5 % do total de processos; e que o somatório dos processos devolvidos pelos 3 analistas equivale a 8,5 % do total de processos. Ora, se a questão nos pede a probabilidade de que um processo escolhido ao acaso tenha sido encaminhado para X, sabendo que foi devolvido, então temos a proporção:0,015/0,085 = 3/17
  • Pelo enunciado x ficou com 30% dos processos, y com 45% e z com os restantes 25%. Foi acrescentado ainda que x teve devolvidos 5% dos processos recebidos, y 10% e z 10%. 

    Quer dizer que do total dos processos, 1,5% dos processos foram devolvidos por x (0,30 * 0,05), y 4,5% e z 2,5%. Somando temos que 8,5% dos processos foram devolvidos. 

    A questão quer saber, se ele foi devolvido, qual a probabilidade de ter sido encaminhado para x? 

    1,5/8,5 = 3/17 

    Gabarito B 

  • Imagine que temos 100 processos. Portanto, 30 foram encaminhados para X, 45 para Y e 25 para Z.

                   X devolveu 5% dos 30 processos que recebeu, isto é, devolveu 5% * 30 = 1,5 processos.

                   Y devolveu 10% dos 45 processos que recebeu, ou seja, 4,5 processos.

                   Z devolveu 10% dos 25 processos que recebeu, ou seja, 2,5 processos.

                   Ao todo, 1,5 + 4,5 + 2,5 = 8,5 processos são devolvidos. Destes, 1,5 são devolvidos por X.

    Assim, sabendo que um processo foi devolvido, a chance de ele ter sido encaminhado para X é:

  • Total devolvido = 1,5 + 4,5 + 2,5 =8,5

    1,5/8,5 =  15    =  3

                     85       17  


ID
73120
Banca
FGV
Órgão
SEFAZ-RJ
Ano
2009
Provas
Disciplina
Estatística
Assuntos

Um torneio será disputado por 4 tenistas (entre os quais A e B) de mesma habilidade, isto é, em qualquer jogo entre dois dos quatro jogadores, ambos têm a mesma chance de ganhar.

Na primeira rodada, eles se enfrentarão em dois jogos, com adversários definidos por sorteio. Os vencedores disputarão a final. A probabilidade de que o torneio termine com A derrotando B na final é:

Alternativas
Comentários
  • 1) Jogos da primeira rodada : C4,2 = 6 jogos possíveis => nessa fase, A não pode jogar com B e, por consequencia, C não pode jogar com D. Com isso, para conseguirmos o resultado final, somente há 4 jogos viáveis. 4/6

    2)A e B devem ganhar seus jogos, logo 1/2 de chance para cada

    3) A deve ganhar de B na final, 1/2

    4/6 * (1/2)3 = 1/12
        
  • Gente achei esse comentário mais didático:

    Tenistas: 
    A, B, C e D

    Jogos possíveis: 

    AB e CD
    AC e BD
    AD e BC


    Para termos A e B na final vemos que só as duas últimas possibilidades são viáveis, logo: P = 2/3

    Agora, vamos trabalhar as probabilidades de A e B ganharem seus jogos e, ainda, A ganhar de B na final.

    P(A avançar pra final) = 1/2
    AC e BD
    AD e BC


    P(B avançar pra final) = 1/2
    AC e BD
    AD e BC


    P(A ser campeão) = 1/2
    A e B na final

    P(total) = (2/3).(1/2).(1/2).(1/2) = 1/12
  • RESOLUÇÃO:

    Chamemos de C e D os outros dois tenistas. Para que A e B disputem a final entre si, é preciso que na primeira rodada eles não joguem um contra o outro, mas sim contra C ou D. Isto é, precisamos ter:

    A x C    e            B x D

    ou

    A x D    e            B x C

    A probabilidade de que o adversário de A seja C ou D, e não B, é de 2 em 3 possibilidades, isto é, 2/3. Com isto, automaticamente o adversário de B será o outro (C ou D).

    A probabilidade de A ganhar seu jogo na primeira rodada é de ½, sendo também esta a probabilidade de B ganhar o seu jogo.

    Por fim, a probabilidade de A vencer B na final também é de ½. Ao todo, para cumprir o exigido pelo enunciado, é preciso que tudo isto ocorra:

    - A enfrente C ou D na primeira rodada (de modo que B enfrentará D ou C);

    - A vença seu jogo;

    - B vença seu jogo;

    - A vença B na final.

    Para tudo isto ocorrer, a probabilidade é de:

    P = (2/3) x (1/2) x (1/2) x (1/2) = 1/12

    Resposta: E


ID
73690
Banca
FGV
Órgão
SEFAZ-RJ
Ano
2008
Provas
Disciplina
Estatística
Assuntos

Os jogadores A e B se encontram para jogar uma partida de tênis em no máximo cinco sets, na qual será vencedor aquele que primeiro ganhar três sets.

Por exemplo, partidas terminadas poderão ter como resultado: AAA, AABA, BABAB, etc. Então, o número de possíveis resultados para uma partida terminada é:

Alternativas
Comentários
  • Está questão pode ser resolvida através do Diagrama de Árvore, pois existem sequências de tamanhos variados, vamos a ele: 1º 2º 3º 4º 5º (RESULTADOS) -- A (A,A,A) | -- A -| -- A (A,A,B,A) | | | | -- B -| | | -- A (A,A,B,B,A) | -- B -| | -- B (A,A,B,B,B) | A -| | -- A (A,B,A,A) | | | -- A -| | | | -- A (A,B,A,B,A) | | -- B -| | | -- B (A,B,A,B,B) -- B -| | -- A (A,B,B,A,A) | -- A -| | | -- B (A,B,B,A,B) -- B -| | -- B (A,B,B,B)Logo acima se encontra a primeira parte do Diagrama de Árvore, como a outra parte é simétrica a ele, conclui-se que a quantidade de resultados possíveis é:(20) -> alternativa D
  • Uma maneira mais fácil de resolver esta questão é através da permutação com repetição, considerando que temos que arrumar 3 letras A (vitória do primeiro jogador) e 2 letras B (vitórias do segundo jogador) e depois fazer o caso inverso, isto é, arrumar 3 letras B (vitórias do segundo jogador)  e 2 letras A (vitória do primeiro jogador). Este é um caso de permutação com repetição das letras. O enunciado já deu a dica que qual seria o método mais rápido para resolver a questão.
     Deste modo, temos:

    2 x (5!/ 2!3!) = 2 x 10 = 20 possibilidades.
  • Na verdade, não é um exemplo de permutação c/ repetição (onde a fórmula é C n+p-1,p), Alexandre. 

    Para se achar a solução da questão, o entendimento correto é permutar (s/ reposição) os 5 sets em 3 vitórias p/ A e para B: C5,3 = 5!/2!3! = 10 x 2 = 20.

    (o cálculo é o mesmo)

    Abs!


  • Se usarmos a permutação dos 5 sets teremos, dentre os resultados: AAABB e BBBAA, que não são possíveis pois quem ganha os primeiros 3 sets já vence a partida, neste caso as formas corretas deveriam ser AAA e BBB, ao invés, AAABB e BBBAA.

    Considerando a vitória de A:

    Permutação de AAABB, considerando a repetição de 3 A's e 2 B's: P (5, 3 e 2) = 5! / (3! * 2!) = 10 . Porém, precisamos subtrair um resultado que não é possível (AAABB), conforme explicado acima, com isso teremos 10 -1 = 9 resultados possíveis. O mesmo serve para a vitória de B, que terá 9 resultados possíveis, pois excluímos BBBAA (totalizando 9 + 9 = 18 resultados até aqui). Além disso, devemos considerar as duas situações de vitória nos três primeiros sets (AAA e BBB), totalizando mais 2 resultados possíves e, portanto, o total de 18 + 2 = 20 resultados

  • Condição 1: Jogador A GANHA

    Para saber quantas combinações possíveis usamos combinação. Então, temos

    Ca5,3 = 5! / ( 3! * ( 5! - 2! ) ) => Ca5,3 = 10.

    Ou seja, há 10 possíveis maneiras do jogador A ganhar.

    Condição 2: Jogador B GANHA

    Para saber quantas combinações possíveis usamos combinação. Então, temos

    Cb5,3 = 5! / ( 3! * ( 5! - 2! ) ) => Cb5,3 = 10.

    Ou seja, há 10 possíveis maneiras do jogador B ganhar.

    Finalmente,

    Ct = Ca + Cb

    Ct = 10 + 10

    Ct = 20

  • Aqui o melhor é listar todos os resultados possíveis. Para vitória de A, temos as seguintes possibilidades:

    AAA

    AABA

    ABAA

    BAAA

    AABBA

    ABABA

    ABBAA

    BABAA

    BBAAA

    BAABA

    Ao todo temos 10 resultados possíveis onde A ganha, pois leva 3 sets. Da mesma forma, teremos mais 10 resultados possíveis onde B ganha, totalizando 20 possibilidades de resultado.

    Resposta: D


ID
100204
Banca
FGV
Órgão
SEAD-AP
Ano
2010
Provas
Disciplina
Estatística
Assuntos

Uma urna contém 50 bolinhas idênticas numeradas de 1 a 50. Se quatro bolinhas são aleatoriamente sorteadas com reposição, a probabilidade de que, dos quatro números sorteados, dois sejam pares e dois sejam impares é igual a:

Alternativas
Comentários
  • 4 . 1/16 = 0,25 = 25% (gaba errado), smj!
  • A resposta está correta.C4,2. (1/2)^2 . (1/2)^2 = 0,375.
  • O comentário da Lizinha está correto.

  • Trata-se de uma distribuição binomial, portanto a probabilidade de sair um número par será p = 25/50 = 1/2 e número ímpar q = 25/50 = 1/2. Deste modo, para que saiam 2 números pares e 2 ímpares teremos:

    P = C4,2 . (1/2)^2.(1/2)^2 = 0.375.
  • Utiliza-se distribuição binomial mas para facilitar o cálculo usa-se a análise combinatória:

    ( 4 2) . ( 0,5) ^2 . ( 0,5) ^2 = 6. 0,25 . 0,25 = 0,375.


    Gabarito LETRA C.
  • Veja que temos reposição, ou seja, a probabilidade de tirar uma bolinha par ou ímpar não muda a cada tentativa. Definindo como “sucesso” a retirada de um número par, o nosso objetivo é conseguir k = 2 sucessos em n = 4 tentativas, sabendo que a probabilidade de sucesso em cada tentativa é p = 50% (afinal metade das bolas são pares). Estamos diante de uma distribuição binomial, cuja probabilidade é dada por:

    Resposta: C

  • Também é possível resolver a questão da seguinte maneira:

    Na situação, existem 6 resultados que satisfazem as condições impostas pelo enunciado:

    Par Par Impar Impar

    Impar Impar Par Par

    Par Impar Par Impar

    Impar Par Impar Par

    Par Impar Impar Par

    Impar Par Par Impar

    A probabilidade de qualquer desses resultados é dada por:

    1/2 * 1/2 *1/2 *1/2 = 1/16

    Dessa forma, para satisfazer as condições impostas, temos que multiplicar a probabilidade de cada resultado pelo número de resultados que satisfazem as condições (6):

    1/16 * 6 = 6/16 (37,50%)


ID
124285
Banca
FGV
Órgão
SEFAZ-RJ
Ano
2010
Provas
Disciplina
Estatística
Assuntos

Se A e B são eventos independentes com probabilidades P[A] = 0,4 e P[B] = 0,5 então P[A ∪ B] é igual a:

Alternativas
Comentários
  • Nao esta aparecendo o sinal de uniao no meu PC. 
    p (A U B) = p (A) + P(B) - P(Â^B) = 0,9 - 0,2 = 0,7
  • Se A e B são independentes, então a probabilidade de A intersecção B é igual ao produto das probabilidades de A e B, logo: p(A U B) = 0,4 +0,5 - 0,4*0,5 = 0,7.

     

  • Axioma da Contabilidade:

    P(A U B ) = P(A) + P(B) - P (A^B)

    Como são eventos independente:

    P (A^B) = P(A) x  P(B)

    Logo:


    P(A U B ) = P(A) + P(B) -( 
    P(A) x  P(B) )
    = 0.4 + 0.5 -(0.4x0.5)
    =0.7
  • estou começando agora nessa parte da matéria e por favor, me tirem uma duvida...

     

    se os eventos são independentes, não seria correto 

    P(AUB) = P(A) + P(B)   ?

    já que não há intersecção entre eles, apenas considero ele zero?

  • Pensei o mesmo, Bruno Ali!!

  • Se os eventos são independentes, sabemos que . Assim, 

    Resposta: D 

  • muita gente erra por confundir eventos mutuamente excludentes com eventos independentes.

  • bruno ali abou ali

    Também ja tive essa mesma dúvida, ocorre que em eventos mutuamente excludentes a intersecção é sempre 0. Nos eventos independentes é que precisamos calcular a intersecção.

  • GABARITO LETRA "D"

    Se os eventos são independentes, sabemos que P (A ∩ B) = P (A) x P (B) = 0,4 x 0,5 = 0,2.

    Logo:

    P (A U B) = P (A) + P (B) - P (A ∩ B)

    P (A U B) = 0,4 + 0,5 − 0,2

    P (A U B) = 0,9 - 0,2

    P (A U B) = 0,7

    FONTE: Prof. Arthur Lima.

    "Se não puder se destacar pelo talento, vença pelo esforço".


ID
125677
Banca
ESAF
Órgão
Prefeitura de Natal - RN
Ano
2008
Provas
Disciplina
Estatística
Assuntos

Uma urna contém: 1 bola amarela; 4 bolas azuis; 10 bolas brancas; 15 bolas vermelhas; e 20 bolas pretas. Dado que na primeira extração foi retirada uma bola vermelha, a probabilidade de na segunda tentativa retirar uma bola vermelha, novamente, é:

Alternativas
Comentários
  • Dados do texto:
     1 bola Amarela
     4 bolas Azuis
    10 bolas Brancas
    15 bolas Vermelhas
    20 bolas Pretas

    Espaço Amostral 50 bolas

    Se a primeira bola saiu Vermelha então a proxima bola a sair a amostra é de apenas 49. Então vamos as opções.

    Primeira Vermelha menos 1 das 15
    então
    a segunda vermelha temos 14/49

    a) maior que retirar uma bola branca ou azul.   14/49  - ERRADA - pois é IGUAL
    b) maior que retirar uma bola preta. 20/49  -  ERRADA  - pois é MENOR
    c) menor que retirar uma bola branca.   10/49    - ERRADA - Pois é MAIOR     
    d) menor que retirar uma bola azul.     4/49   - ERRADA - Pois é MAIOR     
    e) menor que retirar uma bola amarela ou branca ou azul. 15/49   CERTO  POIS É MENOR
  • Para essa questão fiz o seguinte raciocínio:

    COR             QTD         %        1º RETIRADA
    Amarela         1     =     2%
    Azul                 4     =     8%
    Branca          10    =   20%
    Vermelha      15    =   30%     1 = 2%  ===>>>> 14 = 28%
    Preta              20    =   40%
    ________________________________
    TOTAL            50    =  100%


    Para facilitar, cada bola corresponde a um percentual de 2%, pois temos 50 bolas = 100%, ao dividir temos 2% por bola.

    Ao retirar uma bola vermelha, retiramos 2% do total, logo ficou 28% dos 30% totais de bolas vermelhas.

    "...a probabilidade de na segunda tentativa retirar uma bola vermelha, novamente, é: "

    a) maior que retirar uma bola branca ou azul.

    Branca = 10 = 20%
    Azul       = 4   =  8%
    TOTAL  = 14 = 28%

    28% = 28%
    igual não maior.


    b) maior que retirar uma bola preta.


    Preta 20 = 40%

    28% < 40%
    menor não maior.


    c) menor que retirar uma bola branca.

    Branca 10 = 20%

    28% > 20%
    maior não menor.


    d) menor que retirar uma bola azul.

    Azul 4 = 8%

    28% > 8%
    maior não menor.


    e) menor que retirar uma bola amarela ou branca ou azul.

    Amarela 1 = 2%
    Branca  10 = 20%
    Azul         4  = 8%
    TOTAL  25 = 30%

    Logo a probabilidade de se retirar uma segunda bola vermelha é de 28%, menor que retirar uma bola amarela, branca ou azul 30%.

    Letra E correta!


  • onde ta falando q é SEM reposiçao? pq eu fiz com reposição e deu gabarito letra A!!

  • Persistência sempre 2 ELE NÃO FALA QUE REPÕE A BOLA, APENAS QUE RETIRA. ENTÃO É PARA CALCULAR SEM REPOSIÇÃO.


ID
129343
Banca
ESAF
Órgão
SUSEP
Ano
2010
Provas
Disciplina
Estatística
Assuntos

Admita que a probabilidade de uma pessoa de um particular grupo genético ter uma determinada doença é de 30%. Um custoso e invasivo exame para diagnóstico específi co dessa doença tem uma probabilidade de um resultado falso positivo de 10% e de um resultado falso negativo de 30%. Considerando que uma pessoa desse grupo genético com suspeita da doença fez o referido exame, qual a probabilidade dela ter a doença dado que o resultado do exame foi negativo?

Alternativas
Comentários
  • - Falso Positivo: quando os exames indicam que a pessoa está doente, mas não está;
    - Falso Negativo: quando os exames indicam que a pessoa está saudável, mas não está.

    Seja, a seguinte notação: D para doentes; P(D) a probabilidade de um grupo ter a doença; P(~D) a probabilidade de um grupo não ter a doença.

    Logo;

    P(D) = 0.3

    Se 0.3 tem a doença, o complemento não tem, assim:

    P(~D) = 0.7 

    O exercício tb fornece as probabilidades condicionais:

    P(Neg / D) = 0.3 "falso negativo"
    P(Pos/~D) = 0.1 "falso positivo"

    Ou seja, no primeiro caso temos a probabilidade de dado que é doente o exame dar negativo, e no segundo dado q é saudável o exame dar positivo.

    Como queremos P(D/Neg) temos q usar a regra de Bayes;

    P(D/Neg) = P(Neg/D)P(D) / (PNeg)

    P(D/Neg) =  0.3 x 0.3 / [P(Neg/D)P(D) + P(Neg/~D)P(~D)]

    P(D/Neg) = 0.09 / [0.3x0.3 + 0.9x0.7] = 0.09/0.72 = 12.5%



    * Só lembrando que como foi dado P(Pos/~D) = 0.1, podemos calcular seu complemento para colocar na fórmula;
    P(Pos/~D) + P(Neg/~D)= 1

    Assim; P(Neg/~D) = 0.9



  • Resp: 9 / 72 = 12.5%
    Sempre me confundo em montar o Bayes. Prefiro pensar fazendo a tabelinha:
      DOENTE NÃO DOENTE  
    exame positivo 70% 10%  
    exame negativo 30% 90%  
           
    Amostra 100 pessoas DOENTE NÃO DOENTE totais
    exame positivo 21 7 28
    exame negativo 9 63 72
      30 70 100
  • Pelo diagrama de árvore:

                                                 


                                                         positivo 0,7

    de ter a doença 0,3 ----- exame<

                                                         negativo 0,3


        

                                                          positivo 0,1

    não ter a doença 0,7------exame<

                                                          negativo 0,9


    Multiplica ter, pelo exame negativo e divide pela multiplicação de ter pelo exame negativo somado com não ter multiplicado pelo exame negativo. Ou seja, o que se procura dividido pelas possibilidades:

    P ( D ) = 0,3 . 0,3 / ( 0,3 . 0,3 ) + ( 0,7 . 0,9 ) = 0,9 / 0,9 + 0,63 = 0,9 / 0,72 = 0,125    x    100 = 12,5 %

  • Pessoal ainda não entendi.  Tenho dificuldade em matemática. Dessa maneira, devido ao meu limitado conhecimento, pensei assim:

    Probabilidade de ter a doença= 30 %

    Probabilidade de resultado falso negativo = 30 %

    Probabilidade de ter a doença e dar resultado negativo= 30 % x 30 %= 9 %


    Alguém poderia contra- argumentar?


    Obrigado

  • Alguém poderia indicar um bom curso sobre probabilidade. Não entendi nada!

  • Sempre em probabilidade a formula será  o que QUERO / TENHO. 

    Quero exame negativo x total da doença (30% x 30%) = 9%                                                                                                                 Tenho serão os 9% + (não doença x (100% - falso positivo)) => 9% + (70% x 90%) = 72%
                                                                          Quero / Tenho = 9% / 72% = 12,5%
  • O que pega nessa questão é a INTERPRETAÇÃO:

    Falso positivo: pessoa não está doente e o exame diz que ela está (incorretamente)

    Falso negativo: pessoa está doente e o exame diz que ela não está (incorretamente)

     

    Total: 100 pessoas

     

    70 não-doentes (70% do total)

    → 10% de 70 = 7 falso positivo 

    → 90% de 70= 63 negativo (de fato não estão doentes)

     

    30 doentes (30% do total)

    → 30% de 30 = 9 falso negativo (exame deu negativo, mas estão doentes)

    → 70% de 30 = 21 positivo 

     

    p = doentes negativos/ total negativos

    p = 9/72

    p= 0.125

  • Questão esquisita, pois para mim, negativo= falso positivo, mas a questão considerou o negativo= falso negativo. Essas interpretações são fodas...

  • Veja que há 30% de chance da pessoa efetivamente ter a doença, e 70% de chance dela não ter a doença.

           Um resultado falso negativo ocorre quando a pessoa tem a doença, mas o exame indica que a pessoa não a tem. Já um falso positivo ocorre quando a pessoa não tem a doença, mas o exame indica que a pessoa a tem.

           Assim, o resultado do exame pode dar negativo em 2 casos:

    - a pessoa ter a doença (probabilidade = 30%) e o resultado do exame for der negativo (isto é, ocorrer um falso negativo à probabilidade = 30%).

           As chances disso acontecer são P= 30% x 30% = 9%

    - a pessoa não ter a doença (probabilidade = 70%), e o diagnóstico dado pelo exame for correto (isto é, não ocorrer um falso positivo à probabilidade = 1 – 10% = 90%).

           As chances disso acontecer são P = 70% x 90% = 63%.

           Ou seja, no TOTAL, a chance de o resultado do exame dar negativo é dada pela soma de 9% + 63% = 72%. Desses 72%, apenas em 9% dos casos a pessoa efetivamente tem a doença. Portanto, as chances de a pessoa ter a doença, mesmo o exame dando resultado negativo, são:

    P = favoráveis/total = 9% / 72% = 0,125 = 12,5%

    Resposta: E


ID
129352
Banca
ESAF
Órgão
SUSEP
Ano
2010
Provas
Disciplina
Estatística
Assuntos

Considere um grupo de 15 pessoas dos quais 5 são estrangeiros. Ao se escolher ao acaso 3 pessoas do grupo, sem reposição, qual a probabilidade de exatamente uma das três pessoas escolhidas ser um estrangeiro?

Alternativas
Comentários
  • ___ ___ ___ => ( 5/15 x 10/14 x 9/13 ) x 3 = 45/91
  • Tem-se 5 estrangeiros e 10  não estrangeiro = 15 pessoas.

    A probabilidade de se encontrar exatamente 1 estrangeiro é:

    1ª retirada: (5/15) (estrangeiro) * (10/14) * (9/13) = 450/2730
    2ª retirada: (10/15)  * 5/14 (estrangeiro) * (9/13) = 450/2730
    3ª retirada: (10/15)  * (9/14) *  (05/13) (estrangeiro)= 450/2730

    Assim, calcula-se o MMC e se tem: (450 + 450 + 450)/2730 = 45/91
  • Total: 15 pessoas -> escolhendo: 3 pessoas

    C (15, 3) = 455
    5 estrangeiros -> escolhendo 1

    C (5, 1)  = 5
    10 não estrangeiros -> escolhendo 2

    C (10, 2) = 45
    5*45 / 455 = 225 / 455 = 45 / 91.
  • P = [(C5,1*C10,2) / C3,15]

  • 5 estrangeiros

    10 não estrangeiros

    15 total

    P(EXX) = (5/15).(10/14).(9/13) = 15/91

    Como o estrangeiro pode ser o primeiro, o segundo ou o terceiro selecionado:

    P(EXX) = 3.15/91 = 45/91



  • Também dá pra resolver usando distribuição hipergeométrica:

    C 5,1 x C(15 - 5, 3 - 1) / C 15, 3

  • Gabarito: Letra A

    1) O número de formas de se escolher 3 pessoas em um grupo de 15, sem reposição, é C(15,3) = 455.

    2) Para formar grupos com exatamente 1 estrangeiro e 2 brasileiros, temos 5 possibilidades de escolha do estrangeiro e C(10,2) = 45 formas de escolher os brasileiros. Ao todo, temos 5 x 45 = 225 formas de escolher 1 estrangeiro e 2 brasileiros.

    3) Portanto, a chance de formar grupos dessa forma é: P = favoráveis/total = 225 / 455 = 45/91


    Fonte:
    ESTRATÉGIA CONCURSOS

  • Essa questão deveria ser anulada, pois esse valor 45/91 seria COM REPOSIÇÃO!!!!

     

    O valor SEM REPOSIÇÃO é:

     

    5/15 * 10/14 * 9/13 = 15/91

     

    ou

     

    (C5,1 * C10,1 * C9,1) / (C15,1 * C14,1 * C13,1) = 15/91

  • O número de formas de se escolher 3 pessoas em um grupo de 15, sem reposição, é C(15,3) = 455.

           Para formar grupos com exatamente 1 estrangeiro e 2 brasileiros, temos 5 possibilidades de escolha do estrangeiro e C(10,2) = 45 formas de escolher os brasileiros. Ao todo, temos 5 x 45 = 225 formas de escolher 1 estrangeiro e 2 brasileiros.

           Portanto, a chance de formar grupos dessa forma é:

    P = favoráveis/total = 225 / 455 = 45/91

    Resposta: A


ID
172990
Banca
FCC
Órgão
MPU
Ano
2007
Provas
Disciplina
Estatística
Assuntos

O tempo necessário para um medicamento contra dor fazer efeito segue um modelo com densidade Uniforme no intervalo de 5 a 15 (em minutos). Um paciente é selecionado ao acaso entre os que tomaram o remédio. A probabilidade do medicamento fazer efeito em até 10 minutos, neste paciente, é

Alternativas
Comentários
  • A função densidade de probabilidade é p(x = t) = 1/(15 - 5) = 1/10 = 0,1. A probabilidade pedida é p(5 <= x <= 10) = (10 - 5)*0,1 = 5*0,1 = 0,5.

    Resposta: c.

    Opus Pi.

  • Resposta correta: 0,5

  • XMAX - XMIN / 2

    Praticamente o calculo da média.


ID
173005
Banca
FCC
Órgão
MPU
Ano
2007
Provas
Disciplina
Estatística
Assuntos

Em uma livraria 4 livros didáticos com defeito foram misturados a outros 16 livros sem defeito. Um professor foi à livraria e escolheu, aleatoriamente, 4 desses livros para presentear seus alunos. A probabilidade de ter escolhido 3 livros com defeito é

Alternativas
Comentários
  • Temos C(20; 4) de escolher 4 livros. Desse total há C(4; 3) de escolher três livros com defeito e C(16; 1) de escolher 1 livro sem defeito. Pelo princípio fundamental da contagem, temos C(4; 3)*C(16; 1) forma de escolher os 4 livros sendo 3 com defeito. Sendo assim, a probabilidade é C(4; 3)*C(16; 1)/C(20; 4).

    Resposta: a.

    Opus Pi.

  • Alguém explica?

  • Gabarito letra B

     

    Segue a resposta para quem não é assinante e não consegue ver a explicação do professor:

     

    Temos C(20; 4) de escolher 4 livros. Desse total há C(4; 3) de escolher três livros com defeito e C(16; 1) de escolher 1 livro sem defeito. Pelo princípio fundamental da contagem, temos C(4; 3)*C(16; 1) forma de escolher os 4 livros sendo 3 com defeito. Sendo assim, a probabilidade é C(4; 3)*C(16; 1)/C(20; 4).

    Resposta: a.

    Opus Pi.


ID
177709
Banca
FCC
Órgão
TRT - 9ª REGIÃO (PR)
Ano
2010
Provas
Disciplina
Estatística
Assuntos

Em uma população suponha que:

? 80% dos adultos do sexo masculino sejam alfabetizados;
? 60% dos adultos do sexo feminino sejam alfabetizados.

A proporção de adultos do sexo masculino e feminino é igual.

Sorteando-se ao acaso e com reposição uma amostra de 3 pessoas desta população, a probabilidade de se encontrar pelo menos uma alfabetizada na amostra é

Alternativas
Comentários
  • Sem perda de generalidade, considere que há 100 pessoas na população. Desse total:

            - 50 são do sexo masculino e 50 do feminino (pois a proporção é igual);

            - Das 50 do sexo masculino, 40 são alfabetizadas (80%) e 10 são analfabetas (o restante).

            - Das 50 do sexo feminino, 30 são alfabetizadas (60%) e 20 são analfabetas (o restante).

    Assim, do total de 100, temos 40 + 30 = 70 alfabetizadas e 10 + 20 = 30 analfabetas.

    A probabilidade de encontrar pelo menos uma alfabetizada é igual 1 - p(as 3 analfabetas).

    p(as 3 analfabetas) = p(primeira analfabeta)*p(segunda analfabeta)*p(segunda analfabeta)

    p(as 3 analfabetas) = (30/100)*(30/100)*(30/100) = 0,027. Portanto, a probabilidade procurada é 1 - 0,027 = 0,973.

    Resposta: d.

    Opus Pi.

  • população de homem = população mulher = x,

    alfabetizado = (0,8x + 0,6x) / (x + x) = 0,7,
    não alfabetizado = 0,3,
    prob de encontrar ao menos um alfabetizado = 1 - prob de encontrar 3 não alfabetizado = 1 - 0,3^3 = 0,973
  • Binominal : sabe-se que 40% são masculinos e alfabetizados e 30% femininos e alfabetizados,pq a questão diz que tem populações iguais,ou seja, 0,5x0,8 bem como 0,5x0,6.

    Logo: chance de nenhum ser alfabetizado:

    C 3,0 x 0,7^0 x 0,3^3 = 0,027

    Pelo menos 1 alfabetizado= 1-0,027= 0,973


ID
177712
Banca
FCC
Órgão
TRT - 9ª REGIÃO (PR)
Ano
2010
Provas
Disciplina
Estatística
Assuntos

A caixa X tem 5 bolas numeradas de 1 a 5 e a caixa Y tem 7 bolas numeradas de 1 a 7. Uma caixa é selecionada ao acaso e desta seleciona-se aleatoriamente uma bola. Se a bola selecionada apresenta um número ímpar, a probabilidade de que ela tenha vindo da caixa Y é

Alternativas
Comentários
  • A probabilidade de escolher qualquer das caixas é 1/2. Para a caixa X a probabilidade de sair um número ímpar é 3/5. Sendo assim, a probabilidade de a caixar ser escolhida ser X e saí ímpar é (1/2)*(3/5) = 3/10.

    De modo análogo, para a caixa Y, a probabilidade de sair ímpar é 4/7. Sendo assim, a probabilidade de a caixar ser escolhida ser Y e saí ímpar é (1/2)*(4/7) = 2/7.

    Se a caixa escolhida é a Y, a probabilidade de o numero ser ímpar é (2/7)/(2/7 + 3/10) = 20/41.

    Resposta: e.

    Opus Pi.

  • X = 1, 2, 3, 4, 5 = ÍMPAR = 3/5

    Y= 1, 2, 3, 4, 5, 6, 7 = ÍMPAR = 4/7

    Probabilidade de ser ímpar = (3/5) + (7/7) = 41/35

    Probabilidade de ser da caixa Y e ímpar = (4/7) / (41/35) = (4/7) x (35/41) = (4/1) x (5/41) = 20/41 >>>> GAB: E


ID
177718
Banca
FCC
Órgão
TRT - 9ª REGIÃO (PR)
Ano
2010
Provas
Disciplina
Estatística
Assuntos

Em um lote de 8 peças há duas defeituosas e 6 boas. Escolhendo-se ao acaso e sem reposição 3 peças do lote, a probabilidade de se encontrar no máximo uma defeituosa é

Alternativas
Comentários
  • p(no máximo uma defeituosa) = p(nenhuma defeituosa) + p(apenas uma defeituosa), sendo:

    p(nenhum defeituosa) = p(primeira boa)*(segunda boa)*p(terceira boa)

    p(nenhum defeituosa) = (6/8)*(5/7)*(4/6) = 5/14

    p(apenas uma defeituosa) = 3*(2/8)*(6/7)*(5/6) = 15/28

    Assim, p(no máximo uma defeituosa) = 5/14 + 15/28 = 25/28.

    Resposta: c.

    Opus Pi.

  • sem reposição: usar hipergeométrica

  • alguém poderia resolver essa questão por Distribuição Binomial, por gentileza?

  • Fiz por combinação: Nenhuma defeituosa C 6,3 = 20 / TOTAL =C8/3 =56

    1 defeituosa: C 2,1 =2 x C 6,2 =15 =30

    20/56+30/56 = 50/56 simplificando por 2 = 25/28


ID
177721
Banca
FCC
Órgão
TRT - 9ª REGIÃO (PR)
Ano
2010
Provas
Disciplina
Estatística
Assuntos

A inspeção para o controle de qualidade de uma firma examinou os itens de um lote que tem n peças boas e m peças defeituosas (n é muito maior do que m). Uma verificação dos primeiros k(k < m ? 1) itens mostrou que todos eram defeituosos. A probabilidade de que, entre os dois próximos itens selecionados ao acaso, dos restantes, pelo menos um seja defeituoso é:

Alternativas
Comentários
  • O total de peças no lote é n + m. Se k desses itens são defeituosos e já foram examinados, restam n + m - k itens, sendo n bons e m - k defeituosos. Dois itens seguintes serão analisados e queremos saber p(pelo menos um seja defeituoso). Podemos afirmar que:

    p(pelo menos um defeituoso) = 1 - p(nenhum defeituoso)

    Mas

    p(nenhum defeituoso) = p(primeiro ser peça boa)*p(segundo ser peça boa).

    Cada uma das probabilidade do segundo membro é:

    p(primeiro ser peça boa) = n/(n + m - k)

    p(segundo ser peça boa) = (n - 1)/(n + m - k -1).

    Observe que após retirar o primeiro, restam n - 1 bons e m - k defeituosos, de forma que o total é n - 1 + m - k = n + m - k - 1.

    Assim,

    p(nenhum defeituoso) = (n/(n + m - k)).((n - 1)/(n + m - k -1)), o que resulta,

    p(pelo menos um defeituoso) = 1 - [(n/(n + m - k)).((n - 1)/(n + m - k -1))]

    Resposta: a.

    Opus Pi.


ID
199414
Banca
CESPE / CEBRASPE
Órgão
MS
Ano
2010
Provas
Disciplina
Estatística
Assuntos

João foi submetido a um teste de laboratório para o diagnóstico de uma doença rara. A probabilidade de essa doença se desenvolver em um indivíduo como o João é igual a 0,001. Sabe-se que esse teste pode resultar em "falso positivo", ou seja, indicar que João possui essa doença, quando na verdade ele não a tem. Ou, o teste pode resultar em "falso negativo", isto é, indicar que João não possui a doença, quando na verdade ele está doente. A probabilidade de o teste resultar em falso positivo é igual a 0,05 e a probabilidade de o teste resultar em falso negativo é igual a 0,02.

Com base nas informações dessa situação hipotética, julgue os itens subsequentes.

Se qualquer indivíduo como João submeter-se ao teste, então a probabilidade de o teste produzir um resultado negativo é superior a 0,94 e é inferior a 0,98.

Alternativas
Comentários
  • Galera, foi como eu resolvi, espero que seja a forma correta e não esteja falando besteira. É o jeito que encontramos para tentar nos virar, haja vista que o QC não faz o mínimo esforço para pôr professores de estatística. Qualquer erro, mandem uma mensagem, pois não sou dono da verdade.

    Bora lá, monstrinhos lindos:

    Tudo que vou jogar agora está no enunciado:

    • Ter doença é 0,1%
    • Não ter doença é 99,9

    Quando você faz o deste e não tem doença, pode ter apenas 2 resultados 99,9%

    1. Resultado errado, falso-positivo, que fala que você tem, mesmo que não tenha 5%
    2. Resultado correto, negativo, porque você não tem de fato 95%

    Quando você faz o teste E tem doença, pode ter também somente 2 resultados 0,1%

    1. Resultado errado, falso-negativo, que fala que você não tem, mas você está ferrado e tem 2%
    2. Resultado correto, positivo, porque você tem mesmo e o teste prestou 98%

    Agora, a questão pede o resultado seja negativo, tendo duas hipóteses:

    • Você tem a doença, mas o resultado errou (falso-negativo), dizendo que você não tem.
    • 0,1% x 2% = 0,002%
    • Você não tem a doença, e o resultado acertou (falou que você não tem de fato)
    • 99,9% x 95% = 94,90%

    Somando os dois casos, temos

    94,902%

    Gab C.

    #pas

  • Galera, foi como eu resolvi, espero que seja a forma correta e não esteja falando besteira. É o jeito que encontramos para tentar nos virar, haja vista que o QC não faz o mínimo esforço para pôr professores de estatística. Qualquer erro, mandem uma mensagem, pois não sou dono da verdade.

    Bora lá, monstrinhos lindos:

    Tudo que vou jogar agora está no enunciado:

    • Ter doença é 0,1%
    • Não ter doença é 99,9

    Quando você faz o deste e não tem doença, pode ter apenas 2 resultados 99,9%

    1. Resultado errado, falso-positivo, que fala que você tem, mesmo que não tenha 5%
    2. Resultado correto, negativo, porque você não tem de fato 95%

    Quando você faz o teste E tem doença, pode ter também somente 2 resultados 0,1%

    1. Resultado errado, falso-negativo, que fala que você não tem, mas você está ferrado e tem 2%
    2. Resultado correto, positivo, porque você tem mesmo e o teste prestou 98%

    Agora, a questão pede o resultado seja negativo, tendo duas hipóteses:

    • Você tem a doença, mas o resultado errou (falso-negativo), dizendo que você não tem.
    • 0,1% x 2% = 0,002%
    • Você não tem a doença, e o resultado acertou (falou que você não tem de fato)
    • 99,9% x 95% = 94,90%

    Somando os dois casos, temos

    94,902%

    Gab C.

    #pas


ID
199417
Banca
CESPE / CEBRASPE
Órgão
MS
Ano
2010
Provas
Disciplina
Estatística
Assuntos

João foi submetido a um teste de laboratório para o diagnóstico de uma doença rara. A probabilidade de essa doença se desenvolver em um indivíduo como o João é igual a 0,001. Sabe-se que esse teste pode resultar em “falso positivo”, ou seja, indicar que João possui essa doença, quando na verdade ele não a tem. Ou, o teste pode resultar em “falso negativo”, isto é, indicar que João não possui a doença, quando na verdade ele está doente. A probabilidade de o teste resultar em falso positivo é igual a 0,05 e a probabilidade de o teste resultar em falso negativo é igual a 0,02

Com base nas informações dessa situação hipotética, julgue o iten subsequente.

Se o teste ao qual João foi submetido der resultado positivo, então a probabilidade de ele estar de fato com a doença é inferior a 0,02.

Alternativas
Comentários
  • Sabendo que o teste deu positivo:

    • Não ter (99,9%) e deu falso-positivo (5%)
    • Ter (0,1%) e o resultado acertou (98%)

    (99,9% x 5%) + (0,1% x 98%) = 5,098%

    Agora, qual a probabilidade de estar doente? 0,1%

    Estar doente/Sabendo que deu positivo

    0,1%/5,098% = 0,0196

    Logo, deu bom. Gab Certo

  • uma dúvida sobre a resolução do colega. Não estaria faltando mais uma possibilidade no espaço amostral? que seria ele ter a doença(0,1%) e o resultado deu falso-positivo(5%)

    alguém pra conceder uma luz


ID
199420
Banca
CESPE / CEBRASPE
Órgão
MS
Ano
2010
Provas
Disciplina
Estatística
Assuntos

João foi submetido a um teste de laboratório para o diagnóstico de uma doença rara. A probabilidade de essa doença se desenvolver em um indivíduo como o João é igual a 0,001. Sabe-se que esse teste pode resultar em “falso positivo”, ou seja, indicar que João possui essa doença, quando na verdade ele não a tem. Ou, o teste pode resultar em “falso negativo”, isto é, indicar que João não possui a doença, quando na verdade ele está doente. A probabilidade de o teste resultar em falso positivo é igual a 0,05 e a probabilidade de o teste resultar em falso negativo é igual a 0,02.


Com base nas informações dessa situação hipotética, julgue o item.


Se quatro indivíduos que possuem essa doença forem selecionados ao acaso e submetidos ao referido teste de laboratório, e se os resultados forem independentes entre si, então a probabilidade de ocorrerem exatamente dois resultados negativos e dois resultados positivos é inferior a 0,005.

Alternativas
Comentários
  • Binomial, lindezas:

    C4,2 x (2%) * (2%) * (98%) * (98%) = 0,0023

    Gab C


ID
199423
Banca
CESPE / CEBRASPE
Órgão
MS
Ano
2010
Provas
Disciplina
Estatística
Assuntos

Um laboratório farmacêutico produz certo medicamento em três locais diferentes: A, B e C. Do total produzido, 40% têm origem em A; 35% em B e o restante, 25%, tem origem em C. As probabilidades de que haja defeitos no produto final variam segundo o local de origem e são iguais a 0,01, 0,02 e 0,03 para os locais A, B e C, respectivamente. A produção desse laboratório é reunida em certo local D para ser vendida, de maneira que os medicamentos são misturados ao acaso, fazendo com que a identificação da sua origem (A, B ou C) seja impossível.

Considerando essa situação hipotética, julgue o item abaixo.

Se um comprador adquire um medicamento defeituoso no local D, é mais provável que sua origem seja de A.

Alternativas
Comentários
  • Convertendo a porcentagem fica melhor. É produzido: A= 40 ; B= 35 ; C= 25. Destes, possuem defeitos:

    1% de A= 0,40;

    2% de B= 0,70;

    3% de C= 0,75.

    Logo, se um comprador adquire um medicamento defeituoso no local D, é mais provável que sua origem seja de C.


ID
202282
Banca
FCC
Órgão
SEFAZ-SP
Ano
2010
Provas
Disciplina
Estatística
Assuntos

O total de funcionários em uma repartição pública é igual a 6. João e sua esposa trabalham nesta repartição em que será formada uma comissão de 3 funcionários escolhidos aleatoriamente. A probabilidade de que no máximo um deles, João ou sua esposa, faça parte da comissão é

Alternativas
Comentários
  • Comentário objetivo:

    O número total de possibilidades para a formação da comissão é igual a:

    C6,3 = 6! / 3! 3! = 20 casos

    Podemos calcular os casos não desejados. Não desejamos que ambos, João e sua esposa, participem da comissão. Se João e sua esposa participarem da comissão, temos que escolher mais um pessoa dentre 4 restantes.

    Isso acontece em:

    C4,1 = 4! / 1! 3! = 4 casos

    Esses são os casos não desejados. Como o total é 20, desejamos 20 – 4 = 16 casos.

    Assim, a probabilidade pedida é 16/20 = 4/5 (GABARITO D)  

  • Gabarito: Letra D

    1) O total de comissões com 3 funcionários que podem ser formadas a partir de um grupo de 6 funcionários é dada pela combinação de 6, 3 a 3:
    C(6,3) = 20

    2) Dessas, estamos interessados apenas nas que tenham, no máximo, ou João ou sua esposa. Isto é, elas podem ter apenas João, apenas a esposa ou nenhum deles.

    3) Podemos resolver esse problema calculando quantas comissões podem ser formadas incluindo tanto João quanto sua esposa. Neste caso, já temos 2 das 3 pessoas da comissão escolhidas. Temos ainda 4 pessoas disponíveis para a última vaga restante, isto é, 4 possibilidades.

    4) Se existem 4 possíveis comissões incluindo João e também sua esposa, então o número de comissões que tenha, no máximo, um deles, é 20 – 4 = 16. Assim, a chance de obter uma comissão que tenha no máximo 1 deles é: P = 16/20 = 4/5


    Fonte:
    ESTRATÉGIA CONCURSOS

  • Veja que queremos calcular a probabilidade do evento “no máximo um fazer parte”. Ao invés disto, podemos calcular a probabilidade do seu complemento, isto é, “os dois fazerem parte” da comissão. Podemos dizer que:

    P (no máximo um fazer parte) = 100% - P (os dois fazerem parte)

    Para que ambos façam parte da comissão de 3 funcionários, veja que 2 vagas já estão reservadas para o casal. Falta escolher apenas mais 1 elemento, em um total de 6 – 2 = 4 pessoas disponíveis. Isto é, podemos formar 4 comissões diferentes contendo o casal e mais uma pessoa. Esses são os casos FAVORÁVEIS. O TOTAL de comissões com 3 pessoas que podemos formar a partir de um grupo de 6 é dado por C(6,3) = 6x5x4/(3x2x1) = 20. Portanto,

    P(os dois fazerem parte) = 4/20 = 1/5

    Assim, podemos encontrar o que buscamos:

    P (no máximo um fazer parte) = 100% - P (os dois fazerem parte)

    P (no máximo um fazer parte) = 1 – 1/5

    P (no máximo um fazer parte) = 4/5

    Resposta: D

  • Probabilidade: Casos favoráveis/Casos possíves totais

    Casos possíveis totais: C6,3=6!/3!3! = 20 comissões possíveis

    Casos favoráveis:

    Atenção aqui. NO MÁXIMO UM DELES, ou seja, que possua João, sua esposa, ou nenhum deles.

    Retirando João e sua Esposa da lista de funcionários nos sobram 4.

    Comssiões com João: C4,2 = 4!/2!2! = 6(uma vez que João já está nela)

    Comissões com Esposa: C4,2 = 4!/2!2! = 6 (mesmo raciocínio)

    Comissões sem João e sua Esposa: C4,3 = 4!/3!1! = 4

    Somando-se os casos favoráveis = 6 + 6 + 4 = 16

    GABA = 16/20 = 4/5

  • Fiz por binomial:

    1-P(x=2)

    P(knp) = Cn,k * P^(k)*(1-p)^(n-k)

    P(2,6,1/6) = C6,2 * (1/6)^(2)*(1-(1/6))^(6-2) = 0,20

    1 - 0,20 = 0,8 ou seja, 5/4


ID
206236
Banca
FEPESE
Órgão
SEFAZ-SC
Ano
2010
Provas
Disciplina
Estatística
Assuntos

Sejam dois eventos, A e B, mutuamente exclusivos. A probabilidade de ocorrência de A vale 0,2. A probabilidade de ocorrência de B vale 0,4.

Quanto vale a probabilidade de ocorrência do evento A união B?

Alternativas
Comentários
  • UNIÃO significa OU.
    P(A ou B) = P(A) + P(B) - P(A e B)

    P(A e B) para eventos mutuamentes exclusivos é igual a zero.

    Assim: P(A ou B) = 0,2 + 0,4 - 0  =  0,6
  • Eventos excludentes, soma P(A) + P(B) e corre pro abraço.


ID
221488
Banca
FCC
Órgão
TRT - 4ª REGIÃO (RS)
Ano
2009
Provas
Disciplina
Estatística
Assuntos

Considere amostras ordenadas de tamanho 4 com repetição, com escolhas aleatórias tomadas de uma população de tamanho 10. A probabilidade de que nenhum elemento apareça mais de uma vez na amostra é

Alternativas
Comentários
  • (10/10 * 9/10 * 8/10 * 7/10) = 63 / 125


ID
221497
Banca
FCC
Órgão
TRT - 4ª REGIÃO (RS)
Ano
2009
Provas
Disciplina
Estatística
Assuntos

Suponha que se realiza cinco ensaios independentes todos com probabilidade de sucesso igual a 0,3. Seja X a variável aleatória que representa o número de sucessos nesses cinco ensaios e seja Y a variável aleatória que representa o número de sucessos nos três primeiros ensaios. Nessas condições, a probabilidade de Y ser igual a dois, dado que X assumiu o valor três, é igual a

Alternativas
Comentários
  • https://www.tecconcursos.com.br/conteudo/questoes/117211?orgao=trt-4&cargo=analista-judiciario-trt-4-regiao&ano=2009


ID
229303
Banca
FCC
Órgão
TJ-AP
Ano
2009
Provas
Disciplina
Estatística
Assuntos

Dois processadores tipos A e B são colocados em teste por mil horas. A probabilidade de que um erro de cálculo aconteça em um processador do tipo A é de 3%, do tipo B é 2% e em ambos é de 0,3%. A probabilidade de que nenhum processador tenha apresentado erro é igual a

Alternativas
Comentários
  • Comentário objetivo:

    Para resolver essa questão, temos que trabalhar com o conceito de evento complementar. Assim temos:

    DADOS FORNECIDOS PELA QUESTÃO
    P(A ter erro) = 3%
    P(B ter erro) = 2%
    P(A ter erro e B ter erro) = 0,3%

    APLICANDO A FÓRMULA DO "OU"
    A probabilidade de que algum dos processadores tenha erro é calculada pela fórmula do "OU", da seguinte fórmula:

    P (A ou B) = P(A) + P(B) - P(A e B)
    P (A ter erro ou B ter erro) = P(A ter erro) + P(B ter erro) - P(A ter erro e B ter erro)
    P (A ter erro ou B ter erro) = 3% + 2% - 0,3%
    P (A ter erro ou B ter erro) = 4,7%

    UTILIZANDO O CONCEITO DE EVENTO COMPLEMENTAR
    Como encontramos no passo acima a probabilidade de que algum dos processadores (A ou B) tenha erro, para acharmos a probabilidade de que
    nenhum deles tenha erro devemos aplicar o conceito de evento complementar, da seguinte forma:

    P(A não ter erro e B não ter erro) = 100% - P (A ter erro ou B ter erro)
    P(A não ter erro e B não ter erro) = 100% - 4,7%
    P(A não ter erro e B não ter erro) = 95,3%
    P(A não ter erro e B não ter erro) = 0,953 (GABARITO A)
  • Acho que essa questão não tem resposta:

    P(A não ter erro e B não ter erro)= 100%- [ P(A ter erro) + P(B ter erro) + P(A e B terem erros)]=1-[0,03+0,02+0,003]=0,947

ID
229306
Banca
FCC
Órgão
TJ-AP
Ano
2009
Provas
Disciplina
Estatística
Assuntos

Dos 8 caminhões de entrega de uma loja de departamento, três emitem excesso de poluentes. Selecionados aleatoriamente, para a inspeção, 4 dos 8 caminhões, a probabilidade dessa amostra incluir exatamente 2 caminhões que emitem excesso de poluentes é

Alternativas
Comentários
  • Dos 8 caminhões 3 emitem excesso de poluentes. Consequentemente os outros 5 não emitem. Se queremos escolher 4 sendo que exatamente 2 emitem excesso, então os outros 2 necessariamente não devem emitir excesso. Sendo assim, nossa escolha dos quatro será da seguinte forma:

    1) escolher 2 dos 3 que emitem excesso; e

    2) escolher 2 dos 5 que não emitem excesso.

    No caso 1) há C(3;2) = 3 maneiras de escolher e no caso 2) há C(5;2) = 10. Assim, há 3*10 = 30 maneiras de escolher os 4 carros, sendo exatos 2 emitentes de excesso.

    Como temos C(8;4) = 70 maneiras de escolher 4 carros quaisquer, então a probabilidade pedida é 30/70 = 3/7.

    Resposta: d.

    Opus Pi.

  • GABARITO: Letra D

    Probabilidade = Quero/Total

    Dos 4 que vou selecionar, quero 2 poluentes. Logo preciso de outros 2 não poluentes = C(3,2)*C(5,2) = 3*10= 30

    Total de grupos que pode ser formado = C(8,4) = (8*7*6*5)/(4*3*2*1) = 70

    Probabilidade = 30/70 = 3/7


ID
229309
Banca
FCC
Órgão
TJ-AP
Ano
2009
Provas
Disciplina
Estatística
Assuntos

De todas as pessoas que preencheram a declaração do Imposto de Renda em uma comunidade, num determinado ano fiscal, sabe-se que: 10% incluíram deduções que elas sabiam ser ilegais, 5% preencheram a declaração fazendo deduções ilegais por não conhecerem as instruções exatas, enquanto que as demais pessoas a preencheram de forma correta. Sabe-se que 95% das declarações que continham erros propositais e 90% das que continham erros por desconhecimento, foram barradas na malha fina (todas as preenchidas corretamente não foram barradas). Uma declaração é escolhida aleatoriamente dentre as citadas e sabe-se que ela foi barrada na malha fina. A probabilidade da declaração ser de um contribuinte que errou propositalmente é

Alternativas

ID
229312
Banca
FCC
Órgão
TJ-AP
Ano
2009
Provas
Disciplina
Estatística
Assuntos

Um auditor foi contratado para examinar uma coleção de faturas de vendas das quais 10% contêm erros. Ele selecionou, aleatoriamente e com reposição, uma amostra de 4 faturas. A probabilidade de exatamente duas conterem erro é

Alternativas
Comentários
  • prob de exatamente duas com erro = (4 2)*(0,9^2)*(0,1^2) = 0,0486
    obs: (4 2) = combinacao de 4, 2 a 2

  • Poderia explicar melhor essa questão !?. Ainda não consegui entender !


ID
233329
Banca
FUNIVERSA
Órgão
CEB-DISTRIBUIÇÃO S/A
Ano
2010
Provas
Disciplina
Estatística
Assuntos

Um professor de probabilidade aplica a seus alunos
um teste com questões de múltipla escolha de quatro
alternativas (A, B, C, D), sendo apenas uma verdadeira. O
discente deve seguir as instruções: ele pode escolher até
quatro alternativas para ganhar três pontos na marcação
certa e perde um ponto por marcação errada.

Se um aluno fez pelo menos uma marcação em uma questão, qual a probabilidade de ele ter obtido 1 ponto nessa questão?

Alternativas

ID
233332
Banca
FUNIVERSA
Órgão
CEB-DISTRIBUIÇÃO S/A
Ano
2010
Provas
Disciplina
Estatística
Assuntos

Um professor de probabilidade aplica a seus alunos
um teste com questões de múltipla escolha de quatro
alternativas (A, B, C, D), sendo apenas uma verdadeira. O
discente deve seguir as instruções: ele pode escolher até
quatro alternativas para ganhar três pontos na marcação
certa e perde um ponto por marcação errada.

Para obter a melhor pontuação por questão, o número de marcações que o aluno deve fazer, em cada uma delas, é

Alternativas

ID
233350
Banca
FUNIVERSA
Órgão
CEB-DISTRIBUIÇÃO S/A
Ano
2010
Provas
Disciplina
Estatística
Assuntos

Amélia vai sair de férias e pediu ao porteiro do seu edifício que regasse suas plantas aos sábados. Se a planta não for regada durante as férias, a probabilidade de sobreviver é de 10%; se for regada, essa probabilidade é de 50%. Amélia perguntou ao síndico sobre a frequência com que o porteiro atende aos pedidos dos moradores e ele lhe disse que, em 25% dos casos, o porteiro se esquece de atender aos pedidos. Quando Amélia voltar de férias, caso ela encontre as plantas mortas, qual a probabilidade de o porteiro ter se esquecido do pedido dela?

Alternativas

ID
233353
Banca
FUNIVERSA
Órgão
CEB-DISTRIBUIÇÃO S/A
Ano
2010
Provas
Disciplina
Estatística
Assuntos

Ivan usa sua calculadora para gerar números aleatórios entre 0 e 3. Ele está interessado em calcular a sua diferença. Qual a probabilidade de que a diferença obtida seja maior que 2?

Alternativas
Comentários
  • Alguém saberia resolver esta questão?

    Obrigado.
  • Pode-se perceber que estamos trabalhando com uma variável aleatória contínua com função distribuição de probabilidade uniforme.

    Deste modo, para 2 números tais que a diferença entre eles seja igual a 2 (b-a =2), a probabilidade seria a área do retângulo: P = (1/3).(b-a) = 2/3.
    Sobrando então os casos nos quais b-a>2 e b-a<2, tal que a probabilidade para esses 2 casos seria 1 - 2/3 =1/3. Assim, como queremos o caso onde b-a>2, a probabilidade teria que ser menor do que 1/3, o que eliminaria os itens C, D e E.

    Entretanto, não sei como chegar a resposta correta que seria a letra A (1/9) ou B (2/9).
  • Os números aleatórios são entre 0 e 3, assim temos 9 possibilidades: 0 e 3; 1 e 3; 2 e 3; 3 e 3; 0 e 2; 1 e 2; 2 e 2; 0 e 1; 1 e 1. Para que a diferença obtida seja maior que 2, só existe uma possibilidade: 0 e 3, diferença = 3. Logo a probabilidade é 1 em 9, alternativa A.
  • Existem DEZ possibilidades, pois além daquelas citadas pelo colega haveria ainda a chance de as variáveis serem 0 e 0, logo a resposta seria 1/10.
    Questão passivel de recurso.

    abs
  • Vejo 16 possibilidades:
    00
    01
    02
    03
    11
    10
    12
    13
    22
    20
    21
    23
    33
    31
    32
    33
  • Entendo que os números aleatórios vão de 0,0001 a 2,99999.
    Para a diferença ser maior que 2, necessariamente a parte inteira do primeiro número terá que ser 0, e a parte inteira do segundo terá que ser 2.
    As únicas partes inteiras possíveis são 0, 1 ou 2.
    Assim, a probabilidade do primeiro ter parte inteira 0 é de 1/3, e do segundo ter parte inteira 2 é também 1/3.
    A probabilidade final é: P = 1/3 * 1/3 = 1/9 (letra A)

ID
243667
Banca
CESGRANRIO
Órgão
Petrobras
Ano
2010
Provas
Disciplina
Estatística
Assuntos

Em um posto de combustíveis entram, por hora, cerca de 300 clientes. Desses, 210 vão colocar combustível, 130 vão completar o óleo lubrificante e 120 vão calibrar os pneus. Sabe-se, ainda, que 70 colocam combustível e completam o óleo; 80 colocam combustível e calibram os pneus e 50 colocam combustível, completam o óleo e calibram os pneus. Considerando que os 300 clientes entram no posto de combustíveis para executar uma ou mais das atividades acima mencionadas, qual a probabilidade de um cliente entrar no posto para completar o óleo e calibrar os pneus?

Alternativas
Comentários
  • Teoria dos Conjuntos, começa a montar do 50 (que é comum para os 3), depois vai subtraindo.

    Combustível: 210 - 50 - 30 - 20 = 110 (só combustível)
    Pneus: 120 - 50 - 30 = 40
    Óleo: 130 - 50 - 20 = 60

    50: combust. + pneus + óleo
    30: combust. + pneus (80 - 50)
    20: combust. + óleo (70 - 50)


    Deste total teremos: 50 + 20 + 30 + 110 + 60 + 40 = 310, subentende-se que pneus + óleo seria 10, pois a quant. de clientes é 300.

    Por isso, calibrar pneus + completar óleo = 60 clientes
    300 clientes .....................100 %
    60 clientes ....................... x          X = 20 resp. letra B
  • Não entendo, se somente 40 clientes calibraram pneus, como pode calibrar pneus e completar óleo totalizar 60 clientes?
  • Cálculo simples e lógico
    § Se 130 vão completar o óleo
    § Se 120 vão calibrar os pneus
    § Se 300/hora

    Então:

    120+130 = 250 fazem os 2 serviços

    300/250 = 1,20 ou 0,20

    letra B
  • 1) Encontrar quais clientes apenas completaram óleo e calibraram pneus:

    210+130+120-70-80-x=300
    x=10 clientes que APENAS completaram óleo e calibraram pneus.

    2) Somar os que apenas completaram o óleo e calibraram pneus com aqueles que utilizaram todos os serviços.

    10+50=60 TOTAL de clientes que calibraram pneus e completaram o óleo

    3) Calcular a porcentagem TOTAL de clientes que completaram o óleo e calibraram os pneus:

    60/300 = 20%
  • Temos três eventos mutuamente exclusivos. Se chamarmos de A o evento "combustível", de B o evento "óleo" e de C o evento "pneus", ao aplicarmos a regra da adição para três eventos (Diagrama de Venn), temos:

    • P (A ou B ou C) = P (A) + P (B) + P (C) - P (A e B) - P (A e C) - P (B e C) + P (A e B e C), onde:

    P (B e C) corresponde ao total de clientes que entram no posto para completar o óleo e calibrar os pneus;

    • Assim:
    300 = 210 + 130 + 120 - 70 - 80 - P (B e C) + 50

    • Logo:
    P (B e C) = 60 clientes;

    • Portanto:
    P (E) = frequencia do evento / frequencia total = 60/300 = 0,2

     
  • Pessoal, vmaos pela fórumla que é muito simples:
    A+B+C-(AUB+AUC+BUC)+A∩B∩C = Total
    210+130+120-(80+70+x)+50=300
    x=60

    p = 60/300 = 20%
  • Cláudio Martins, já tinha chegado aos 10 clientes que calibraram o pneu e botaram óleo. Mas não estava entendendo da onde vinha o 60 rs.

    Acontece que temos que adicionar a esses 10 clientes, os 50 clientes que fizeram os tres serviços, pois eles tbm calibraram o pneu e botaram oleo.

  • muito mal elaborada!!


ID
263926
Banca
CESGRANRIO
Órgão
Petrobras
Ano
2011
Provas
Disciplina
Estatística
Assuntos

Um jogo consiste em lançar uma moeda honesta até obter duas caras consecutivas ou duas coroas consecutivas. Na primeira situação, ao obter duas caras consecutivas, ganha-se o jogo. Na segunda, ao obter duas coroas consecutivas, perde-se o jogo. A probabilidade de que o jogo termine, com vitória, até o sexto lance, é

Alternativas
Comentários
  • Total de casos = 2^6 = 64

    Casos de empate ----> CKCKCK e KCKCKC ----> 2 casos (K = cara, C = coroa)

    Sobram 62 casos, sendo 31 de vitória e 31 de derrota

    Pv = 31/64
  • Legenda:
    C: cara
    K: coroa

    Ganha-se obtendo duas caras consecutivas, dessa forma, eu ganho tirando:
    CC = 1/2*1/2 = 1/4
    KCC = 1/2*1/2*1/2 = 1/8
    CKCC = 1/2*1/2*1/2*1/2 = 1/16
    KCKCC = 1/2*1/2*1/2*1/2*1/2 = 1/32
    CKCKCC = 1/2*1/2*1/2*1/2*1/2*1/2 = 1/64

    somando tudo temos 31/64.
  • Para os 6 lançamentos da moeda, quantos são os casos possíveis?   Ora, temos 6 lançamentos, e cada um deles tem dois resultados possíveis (K ou C). Aplicando o princípio fundamental da contagem, onúmero de casos possíveis é:   2 * 2 * 2 * 2 * 2 * 2 = 64   Tudo certo até aqui?   Continuando.    Há situações em que não ocorre nem a vitória nem a derrota. Tais situações correspondem aos casos em que cara e coroa se intercalam. São dois casos desse tipo:   C, K, C, K, C, K K, C, K, C, K, C   Tínhamos então 64 possíveis.   Em 2 desses 64 casos, não temos nem vitória nem derrota.   Sobram 64 - 2 = 62.   Nesses 62 que sobraram, ou temos vitória ou temos derrota.   Como a chance de K ou C é a mesma (50% para cada lado), a chance de derrota é igual à chance de vitória. Assim, metade dos 62 casos correspondem a vitórias e metade correspondem a derrotas.   Disto resulta que em 62 / 2 = 31 casos nós temos vitória até o sexto lançamento. Em outras palavras, são 31 casos favoráveis.   A probabilidade é dada pela relação entre número de casos favoráveis e número de casos possíveis. Ficamos com:   P = 31 / 64
  • Questão muito interessante. O cálculo combinatório direto é um pouco difícil, mas felizmente podemos obter a resposta indiretamente. Qualquer seja o número de lances, o jogo só não termina em 2 possibilidades: uma sequência alternada se iniciando em cara, ou coroa. Vemos então que, em 6 lances, a chance do jogo não terminar é 2/64. (pois 64=2^6 é o número total de possibilidades em 6 lances). Das possibilidades restantes, é fácil ver que metade termina em vitória e metade em derrota, ou seja, o número de possibilidades de vitória é 62/2=31, donde P_vitória = 31/64. 

    (B) 31/64 (Resposta correta)
  • Explicando a fórmula de probabilidade a qual é dada pela relação entre número de casos favoráveis / número de casos possíveis; 
    Se temos duas possibilidade (K ou C) a cada arremesso...;
    Considerando seis arremessos, significa dizer que existem 2 (elevado) a 6 (2^6)=64 chances de vitória ou derrota (casos possíveis)...; 
    Descartamos duas situações em que não saiu coroa, logo 64 - 2 = 62... Isto significa que temos 31 possibilidades de K + 31 de C,  O problema pede apenas as chances de vitória consideraremos apenas 31 K (casos favoráveis) dentre 64 chances no total(casos possíveis). Na relação de probabilidade conforme a fórmula descrita temos P=31/64.   

  • entendi foi nada

  • São 64 possibilidades.
    Apenas dois empates: KCKCKC ou CKCKCK
    Sobram 62 possibilidades, sendo 31 pra cada.

    31/64 é a resposta.


ID
269557
Banca
CESPE / CEBRASPE
Órgão
TRE-ES
Ano
2011
Provas
Disciplina
Estatística
Assuntos

Julgue os itens a seguir, relativos ao cálculo de probabilidades.

Considere que um jogador pague R$ 1,00 para retirar aleatoriamente duas bolas de uma urna que contém dez bolas numeradas de 1 a 10 e, que, se ele retirar as duas bolas numeradas com 1, 2 ou 3, ele ganhe R$ 10,00. Nesse caso, a expectativa de ganho desse jogador será positiva somente se as bolas forem retiradas com reposição.

Alternativas
Comentários
  • com reposicao:
    3/10 * 3/10 = 9/100 (probabilidade de tirar duas bolas numeradas com 1, 2 ou 3)
    lucro = ganho - gasto
    ganho = 9/100 * 10 = 0,90
    logo, lucro = 0,90 - 1 = -0,10
    a banca empregou erroneamente o termo expectativa de ganho.. o correto é expectativa de lucro

     


ID
314278
Banca
FCC
Órgão
TRT - 1ª REGIÃO (RJ)
Ano
2011
Provas
Disciplina
Estatística
Assuntos

Um estudo sobre salários associados ao estado civil dos indivíduos de certa comunidade revelou que a proporção de indivíduos:

I. solteiros é de 0,4.

II. que recebem até 5 salários mínimos é de 0,3.

III. que recebem entre 5 (exclusive) e 10 (inclusive) salários mínimos é de 0,5.

IV. que recebem até 5 salários mínimos entre os solteiros é de 0,3.

V. que são não solteiros dentre os que recebem mais do que 10 salários mínimos é de 0,8.

Um indivíduo é selecionado ao acaso dessa comunidade. A probabilidade de ele ser solteiro e ganhar entre 5 (exclusive) e 10 (inclusive) salários mínimos é

Alternativas
Comentários
  • Solteiro e ganhar ate 5 salarios = 0,4 * 0,3 = 0,12
    Logo não solteiro com igual salario = 0,6 * 0,3 = 0,18
    Não solteiro e ganhar mais de 10 salarios = 0,6 * 0,2 = 0,12
    Solteiro com igual salario = 0,4 * 0,2 = 0,08
    Não solteiro ganhando entre 5 e 10 salarios = 0,6 * 0,5 = 0,3
    Solteiro ganhando igual salario = 0,5 * 0,4 = 0,2: nao tem essa resposta em nenhuma das alternativas

    Observe que a soma de todos os eventos dá 1.

  • O gabarito é a letra A

    Encontrei uma resolução mais clara no Fórum

    Como a questão deu apenas valores relativos (proporções), podemos, sem perda de generalidade, assumir um valor para o total de indivíduos. Para facilitar as contas, suponhamos que seja 100. Montemos a tabela (SM é salário mínino e S é o salário):

    _____________S <= 5 SM______5 SM < S <= 10 SM______S > 10 SM___Total
    solteiros_________A___________________B___________ ______C_______D
    não-solteiros_____E___________________F_______________ ___G_______H
    Total____________M___________________N____________ _____P______100

    A questão quer saber o valor de B/100.

    I. solteiros é de 0,4.

    Isso significa que D = 0,4*100 = 40.

    II. que recebem até 5 salários mínimos é de 0,3.

    Isso significa que M = 0,3*100 = 30.

    III. que recebem entre 5 (exclusive) e 10 (inclusive) salários mínimos é de 0,5.

    Isso significa que N = 0,5*100 = 50.

    IV. que recebem até 5 salários mínimos entre os solteiros é de 0,3.

    Isso significa que A = 0,3*D = 0,3*40 = 12.

    V. que são não solteiros dentre os que recebem mais do que 10 salários mínimos é de 0,8.

    Isso significa que G = 0,8*P. Observe que M + N + P = 100, ou seja, 30 + 50 + P = 100, portanto P = 20. Resta que G = 16.

    Colocando os valores encontrados acima na tabela:

    _____________S <= 5 SM______5 SM < S <= 10 SM______S > 10 SM___Total
    solteiros_________12___________________B__________ _______C_______40
    não-solteiros_____E___________________F_______________ ___16_______H
    Total____________30__________________50___________ _____20______100

    Pela tabela, C + 16 = 20 => C = 4.

    Também temos 12 + B + C = 40 => 12 + B + 4 = 40 => B = 24.

    Portanto, B/100 = 24/100 = 0,24.

ID
314302
Banca
FCC
Órgão
TRT - 1ª REGIÃO (RJ)
Ano
2011
Provas
Disciplina
Estatística
Assuntos

Após o lançamento de um novo modelo de automóvel observou-se que 20% deles apresentavam defeitos na suspensão, 15% no sistema elétrico e 5% na suspensão e no sistema elétrico. Selecionaram-se aleatoriamente e com reposição 3 automóveis do modelo novo. A probabilidade de pelo menos dois apresentarem algum tipo de defeito é

Alternativas
Comentários
  • Como 5% tem ambos os tipos de defeito, então, 15% (20 - 5) tem só defeito na suspensão e 10% (15 - 5) só defeito elétrico. Assim, a probabilidade do item apresentar defeito é igual a 30% (5 + 15 + 10).
    prob (3 com defeito) = 0,3^3 = 0,027
    prob (2 com defeito) = combinacao de (3 2)*(0,3^2)*(0,7) = 0,189
    a soma destas duas ultimas probabilidades responde o que foi solicitado no problema = letra E

     

  • Carlos Eduardo,
    Não entendi  parte final da sua explicação "cominação de 3 2"?
    Poderia por favor detalharmais sua explicação? Obrigado.


  • Raciocinando por Análise Combinatória, pensei o seguinte:

    Excluir os carros que têm os 2 defeitos para saber quantos carros efetivamente saem com defeito
    20% - suspensão                        20% - 5% = 15% - suspensão
    15% - elétrico                                15% - 5% = 10% - elétrico
    5% - suspensão e elétrico                                 5% - suspensão e elétrico
                             Total de carros com defeito = 30%
    Ou seja, se tivermos 100 carros, 30 saem algum defeito e 70 saem perfeitos.

    Ele quer pelo menos 2 com algum defeito de 3 carros retirados, então podem ser:
     D .   D.   D      30/100  .  30/100  .  30/100   
    OU +  
      D.    D.    P   30/100 .  30/100  .  70/100 .   3!/2! (multi por 3 fatorial pq tem q permutar D . D. P entre si)  (dividi por 2 fatorial pq D aparece 2X)

    Fazendo as contas acharemos:
                27/1000    +   189/1000  =  216/1000  = 0,216  ----------- Resp. : E.
       
            


ID
318418
Banca
CESPE / CEBRASPE
Órgão
FUB
Ano
2011
Provas
Disciplina
Estatística
Assuntos

Julgue o item que se segue, referente ao controle estatístico de qualidade.

Em um processo industrial que está sob controle, a probabilidade de erro tipo I deve ser utilizada para se determinar o tempo médio de espera até a observação do primeiro alarme falso.

Alternativas
Comentários
  • erro tipo I = rej h0 dado q h0 é verdadeira = falar que o produto tem defeito quando na verdade ele nao tem = alarme falso

    exemplo:

    h0: leite tem 0,01 de açúcar ou menos
    h1: tem tem mais de 0,01

     


ID
318532
Banca
CESPE / CEBRASPE
Órgão
STM
Ano
2011
Provas
Disciplina
Estatística
Assuntos

Acerca da teoria de probabilidades, julgue os itens subsecutivos.

O número de possíveis retiradas de bolas de uma urna, sem reposição, é menor quando a ordem importa.

Alternativas
Comentários
  • considere uma urna com tres bolas, assim enumeradas: 1, 2 e 3

    suponhamos que eu vá retirar 2 bolas

    sem reposicao, eu posso retirar 2 bolas com o seguinte dispositivo (quando a ordem importa):

    12
    13
    23
    21
    32
    31

    quando a ordem nao importa:

    12
    13
    23

    portanto, quando a ordem importa, temos mais possibilidades

     

  • Errado

    é exatamente a mesmo possibilidade

    Exemplo 1: tenho 30 bolas e quero tirar 3, sem reposição

    1/30*1/29*1/28

    Exemplo2: tenho 30 bolas e quero tirar os números 5, 8 e 12, sem reposição

    1/30*1/29*1/28

  • É só pensar no RLM onde quando a ordem importa a gente usará arranjo, que por sua vez sempre será maior do que quando a ordem não importa, usando- se a combinação nesse ultimo caso


ID
318535
Banca
CESPE / CEBRASPE
Órgão
STM
Ano
2011
Provas
Disciplina
Estatística
Assuntos

Acerca da teoria de probabilidades, julgue os itens subsecutivos.

Se 80% de uma população pertence ao grupo A e 60%, ao grupo B, e sabendo que a interseção entre os grupos A e B não é vazia, então a probabilidade da interseção de A e B é maior ou igual a 0,4.

Alternativas
Comentários
  • 0,8+0,6-1=0,4

  • P(A∩B)= P(A) X P(B)= 0,8 X 0,6= 0,48

  • Correto

    A interseção de A+B é a soma de A+B, subtraindo os 100%.

    Ou seja 40% ou 0,4


ID
318538
Banca
CESPE / CEBRASPE
Órgão
STM
Ano
2011
Provas
Disciplina
Estatística
Assuntos

Acerca da teoria de probabilidades, julgue os itens subsecutivos.

Se X = I(A) é uma função indicadora da ocorrência do evento A, então E(X) = P(A), em que E(X) é o valor esperado de X e P(A), a probabilidade de ocorrência do evento A.

Alternativas

ID
318541
Banca
CESPE / CEBRASPE
Órgão
STM
Ano
2011
Provas
Disciplina
Estatística
Assuntos

Acerca da teoria de probabilidades, julgue os itens subsecutivos.

A distribuição uniforme contínua em [0, 1] é um caso degenerado da distribuição beta.

Alternativas
Comentários
  • Quando a distribuição Beta tem seus parâmetros de forma (a e b) iguais a 1, temos uma Beta degenerada:

    http://www.portalaction.com.br/probabilidades/610-distribuicao-beta


ID
334897
Banca
FGV
Órgão
SEFAZ-RJ
Ano
2011
Provas
Disciplina
Estatística
Assuntos

Quantas combinações existem para determinar o primeiro e o segundo lugares de um concurso com 10 pessoas? (O primeiro e o segundo lugares não podem ser a mesma pessoa).

Alternativas
Comentários
  • Existem 10 alternativas para a primeira colocação, como não pode se repetir há 9 alternativas para a segunda colocação, então:
    10 x 9 = 90
  • Ou pela fórmula do arranjo:

    =10!/(10-2)!=
    =(10x9x8!)/8!=   >>>>> (corta 8! de cima com o 8! de baixo)
    =10x9=90
    --------------------------------
    Um arranjo de n elementos dispostos p a p, com p menor ou igual a n, é uma escolha de p entre esses n objetos na qual a ordem importa. Sua fórmula é dada por

    A(n,p) = n! / (n - p)!

    O exemplo mais clássico de arranjo é o pódio: em uma competição de 20jogadores, quantas são as possibilidades de se formar um pódio com os três primeiros lugares? Note que, neste problema, queremos dispor 20 jogadores em 3 lugares, onde a ordem importa, afinal o pódio formado por João, por Marcos e por Pedro não é o mesmo formado por Pedro, por Marcos e por João. 

    FONTE: 
    http://www.andremachado.org/artigos/440/entenda-a-diferenca-entre-permutacao-arranjo-e-combinacao.html

  • Arranjo
    10! / 8! --> 10 * 9 * 8! / 8! --> 10 * 9 => 90

  • Temos 10 opções para o primeiro lugar e 9 restantes para o segundo lugar, totalizando 10 x 9 = 90 possibilidades.

    Resposta: B

  • Minha contribuição.

    Veja que podemos resolver essa questão sem uso de fórmulas. Temos 10 pessoas disponíveis para a primeira posição e, com isso, sobram 9 pessoas para a segunda colocação, num total de 10x9 = 90 possibilidades.

    Resposta: B

    Fonte: Direção

    Abraço!!!


ID
339598
Banca
COSEAC
Órgão
DATAPREV
Ano
2009
Provas
Disciplina
Estatística
Assuntos

A urna I contém 9 bolas: 3 pretas, 2 brancas e 4 vermelhas. A urna II contém 8 bolas: 4 pretas, 1 branca e 3 vermelhas. A urna III contém 9 bolas: 1 preta, 3 brancas e 5 vermelhas. Escolhe-se uma urna ao acaso e dela extrai-se uma bola também ao acaso, sabendo que a bola sorteada foi branca, a probabilidade de ter vindo da urna II é de:

Alternativas

ID
339616
Banca
COSEAC
Órgão
DATAPREV
Ano
2009
Provas
Disciplina
Estatística
Assuntos

Um dispositivo eletrônico recentemente desenvolvido apresenta vida média de 80 horas. Considerando o comportamento segundo a distribuição exponencial, a probabilidade desse dispositivo durarmais de 100 horas é:

Alternativas
Comentários
  • alpha (A) é o inverso da média.

    ao integrar  ae elevado -ax = -e elevado-ax

    1-(1-e elevado -(100/80)) = e elevado -(100/80)

  • GAB D

    A função densidade de probabilidade de uma distribuição exponencial serve pra indicar valores abaixo de x. A questão quer saber valores acima de x. Basta usar o método complementar.

    Se f(x) = 1 - e^-lambda . x indica valores abaixo

    então f(x) = e^-lambda . x indicará valores acima.

    x da questão = 100

    Para achar o valor do parâmetro lambda: nos valeremos da média que fora dada

    80 = 1/lambda

    lambda = 1/80

    lambda . x = 1/80 . 100 = 100/80. Simplificando -> 5/4


ID
339640
Banca
COSEAC
Órgão
DATAPREV
Ano
2009
Provas
Disciplina
Estatística
Assuntos

O nível de significância de um teste de hipóteses é a probabilidade de:

Alternativas
Comentários
  • Decorebinha. 

     

    Hipótese verdadeira

    Se aceito - ok (fiz certo)

    Se rejeito -  erro tipo 1 (ou alfa) -  O nível de significância representa a probabilidade de Erro Tipo I, ou seja, é a probabilidade de rejeitarmos uma hipótese verdadeira.

     

    Hipótese falsa

    Se aceito - erro tipo 2

    Se rejeito - ok (fiz certo)

     


ID
349990
Banca
CESGRANRIO
Órgão
EPE
Ano
2010
Provas
Disciplina
Estatística
Assuntos

Dado que duas pessoas fazem aniversário no mês de junho, a probabilidade de que ambas tenham nascido no dia 08 de junho é de

Alternativas
Comentários
  • RESPOSTA LETRA C

    • Na matemática o mês tem 30 dias!

     

    • Tem que nascer no dia 08 do mês, então só temos uma opção.

     

    Então temos, 1/30, como são duas pessoas, cada uma vai ter a chance 1/30.

    Agora o detalhe principal da questão!!

    Ela quer que ambas nasça no mesmo dia, a palavra ambas significa multiplicação na matemática.

    1/30.1/30 = 1/900


ID
349993
Banca
CESGRANRIO
Órgão
EPE
Ano
2010
Provas
Disciplina
Estatística
Assuntos

Um dado comum foi alterado para que, quando lançado, a probabilidade de obter o número 6 fosse de 50% e a pro- babilidade de obter qualquer um dos demais números fosse de 10%. Logo, a distribuição de probabilidades so- bre os eventos elementares 1, 2, 3, 4, 5 e 6

Alternativas

ID
350512
Banca
CESPE / CEBRASPE
Órgão
IJSN-ES
Ano
2010
Provas
Disciplina
Estatística
Assuntos

Considerando os conceitos associados a probabilidade e estatística,
julgue os itens de 108 a 116.

Com base no espaço amostral S = {1, 2, 3, 4} é correto afirmar que os eventos A = {2 ,4} e B = {1, 4}, não são independentes.

Alternativas
Comentários
  • ERRADO.

    Dois eventos são independentes quando a ocorrência de um evento não influência a ocorrência do outro evento.

    Por exemplo:

    U = {1, 2, 3, 4}

    { {1,2}, {1,3}, {1,4}, {2,3}, {2,4}, {3,4}}

    Escolher o {2,4} não influência escolher {1,4}.

    Carlos André Botelho

  • Podem ser independentes A∩B = {2,4} x {1,4}

    A∩B = {4}


ID
350515
Banca
CESPE / CEBRASPE
Órgão
IJSN-ES
Ano
2010
Provas
Disciplina
Estatística
Assuntos

Considerando os conceitos associados a probabilidade e estatística,
julgue os itens de 108 a 116.

Considere que de uma urna contendo 2 bolas azuis e 6 bolas brancas retira-se ao acaso uma bola, anota-se sua cor e repõe-se a bola na urna. Em seguida retira-se novamente uma bola da urna e anota-se sua cor. Nessas condições, a probabilidade das duas bolas retiradas serem azuis é 1/4.

Alternativas
Comentários
  • Gabarito E

    Como a questão pede que as duas bolas sejam azuis, então multiplica-se as probabilidades.

    Logo, 2/8 x 2/8 = 1/16

  • Excelente comentário.


ID
350518
Banca
CESPE / CEBRASPE
Órgão
IJSN-ES
Ano
2010
Provas
Disciplina
Estatística
Assuntos

Considerando os conceitos associados a probabilidade e estatística,
julgue os itens de 108 a 116.

Considere que as bolas de uma urna tenham sido divididas em duas caixas, sendo que a caixa I ficou com 3 bolas brancas e 1 bola preta, enquanto a caixa II ficou com 3 bolas brancas e 3 bolas pretas. Em seguida, uma caixa foi escolhida e dela foi sorteada uma bola. Sabendo que a cor da bola sorteada era branca, a probabilidade de ter vindo da caixa I é de 3/8.


Alternativas
Comentários
  • ERRADO.

    Caixa 1 = { B, B, B, P }

    Caixa 2 = { B, B, B, P, P, P }

    Trata-se de uma Probabilidade Condicional: 

    Sabemos que a bola sorteada é branca, então nosso universo não é mais o total e sim a probabilidade de ser branca.

    Branca - caixa 1 => 1/2(prob de escolher caixa 1) x 3/4 (prob de escolher bola branca da caixa 1) = 3/8

    Branca - caixa 2 => 1/2(prob de escolher caixa 2) x 1/2 (prob de escolher bola branca da caixa 1) = 1/4

    Nosso Universo = 3/8 + 1/4 = 5/8

    P condicional: 3/8 / 3/5

    P condicional: 3/5

    Portanto:

    O item é Errado.

  • 3/6 sobre 6/10

  • Difícil explicar sem desenhar, mas vou tentar pela fórmula:

    P(A/B) = P(B/A).P(A) / P(B)

    P(B)= probabilidade de ser branca

    P(B)= prob de ser branca na caixa 1 + prob de ser branca na caixa 2 = 1/2*3/4 + 1/2*3/6 = 5/8

    P(A) = probabilidade de escolher caixa 1 = 1/2

    P(B/A)= probabilidade de escolher a branca dado q escolheu caixa 1 = 3/4

    P(A/B) = (3/4*1/2) / 5/8 ----> 3/8*8/5 = 3/5


ID
350521
Banca
CESPE / CEBRASPE
Órgão
IJSN-ES
Ano
2010
Provas
Disciplina
Estatística
Assuntos

Considerando os conceitos associados a probabilidade e estatística,
julgue os itens de 108 a 116.

A probabilidade de se obter um número menor que 5 no lançamento de um dado, sabendo que o dado não é defeituoso e que o resultado é um número ímpar, é igual a 2/3.

Alternativas
Comentários
  • CORRETA

    dado = 1,2,3,4,5,6

    impares = 1,3,5 ---> total 3

    probabilidade de sair um impar menor que 5 = 2/3


ID
350527
Banca
CESPE / CEBRASPE
Órgão
IJSN-ES
Ano
2010
Provas
Disciplina
Estatística
Assuntos

Considerando os conceitos associados a probabilidade e estatística,
julgue os itens de 108 a 116.

A hipótese nula (H0) é a afirmação feita acerca do valor de um parâmetro populacional e o erro tipo I ocorre quando a hipótese nula é falsa e não é rejeitada.

Alternativas
Comentários
  • Hipótese nula, de fato, tem como base um parâmetro populacional. O erro da questão está na definição do erro tipo 1, pois o conceito é do erro tipo 2.

    ERROS (e poder do teste):

    TIPO 1: Rejeita H0 quando for VERDADEIRA.

    TIPO 2: Aceita H0 quando for FALSA.

    Poder do teste: Rejeita H0 quando for FALSA.

    Gabarito ERRADO.


ID
470428
Banca
CESGRANRIO
Órgão
Transpetro
Ano
2011
Provas
Disciplina
Estatística
Assuntos

A respeito das medidas de tendência central e de dispersão de uma distribuição de probabilidades, verifica-se que a(o)

Alternativas
Comentários
  • A) Não necessariamente moda é maior que média
    B) Não há uma relação exata entre mediana e desvio padrão
    C) Variancia é o desvio padrão elevado ao quadrado
    D) Variancia é uma medida de dispersao
    E) desvio padrão tambem é uma media de dispersão


ID
470437
Banca
CESGRANRIO
Órgão
Transpetro
Ano
2011
Provas
Disciplina
Estatística
Assuntos

A probabilidade de que ocorra o evento X, dado que o evento Y ocorreu, é positiva e representada por P(X/Y). Si- milarmente, a probabilidade de que ocorra Y, dado que X ocorreu, é representada por P(Y/X). Se P(X/Y) = P(Y/X), os eventos X e Y são

Alternativas
Comentários
  • P(Y/X)=P(X interseção Y)/ P(X)

    P(X/Y)=P(X interseção Y)/ P(Y)

    Se P(X/Y) = P(Y/X)
    Logo P(X)=P(Y)

    LOGO SÃO IGUALMENTE PROVAVEIS
    LETRA D

ID
481678
Banca
CESPE / CEBRASPE
Órgão
TRT - 5ª Região (BA)
Ano
2008
Provas
Disciplina
Estatística
Assuntos

Em um local de atendimento ao público chegam, em média,
5 pessoas por hora. Nesse local, há um único servidor que, em
média, atende 10 pessoas por hora. Considerando um modelo fila
simples, sem limite de capacidade, julgue os itens subseqüentes.

A probabilidade de que não haja pessoas na fila em certo horário é superior a 0,4.

Alternativas
Comentários
  • rô = taxa de chegada / taxa de serviço
        = 5 / 10
        = 0,5

    prob de não haver pessoa na fila = 1 - rô = 0,5

     


ID
481681
Banca
CESPE / CEBRASPE
Órgão
TRT - 5ª Região (BA)
Ano
2008
Provas
Disciplina
Estatística
Assuntos

Em um local de atendimento ao público chegam, em média,
5 pessoas por hora. Nesse local, há um único servidor que, em
média, atende 10 pessoas por hora. Considerando um modelo fila
simples, sem limite de capacidade, julgue os itens subseqüentes.

Em determinado horário, a probabilidade de que a fila seja formada por 10 ou mais pessoas é inferior a 0,01.

Alternativas
Comentários
  • probabilidade de n clientes no sistema:

    (1 - rô)*rô^n = 0,0005, pois:

    rô = taxa de chegada / taxa de serviço

    n = 10

ID
481714
Banca
CESPE / CEBRASPE
Órgão
TRT - 5ª Região (BA)
Ano
2008
Provas
Disciplina
Estatística
Assuntos

Texto para os itens de 74 a 80

Em um presídio, há 500 prisioneiros, dos quais 150 são
réus primários e os 350 restantes são réus reincidentes. Entre os
réus reincidentes, há 170 que cumprem penas de cinco anos ou
mais.
Com relação às informações do texto, julgue os itens a seguir.

Se um réu reincidente for escolhido ao acaso, a probabilidade de ele estar cumprindo pena de cinco anos ou mais é superior a 0,65.

Alternativas
Comentários
  • GAB. E

    Probabilidade é: Casos favoráveis / Total de casos

    Da questão extraímos:

    • 350 são reincidentes
    • 170 cumprem pena de cinco anos ou mais

    Nosso total será 350, pois escolhemos, ao acaso, dentre os reincidentes.

    O caso favorável é o que queremos descobrir, ou seja, a probabilidade de estar cumprindo pena de cinco anos ou mais.

    Assim, 170/350 = 17/35 = 0,485 = 48,5% aproximadamente.


ID
481720
Banca
CESPE / CEBRASPE
Órgão
TRT - 5ª Região (BA)
Ano
2008
Provas
Disciplina
Estatística
Assuntos

Texto para os itens de 74 a 80

Em um presídio, há 500 prisioneiros, dos quais 150 são
réus primários e os 350 restantes são réus reincidentes. Entre os
réus reincidentes, há 170 que cumprem penas de cinco anos ou
mais.
Com relação às informações do texto, julgue os itens a seguir.

Ainda com relação às informações do texto, e considerando que três presidiários sejam selecionados aleatoriamente (sem reposição), julgue os itens subseqüentes.

A probabilidade de que a amostra contenha exatamente 1 réu primário e 2 réus reincidentes é inferior a 0,80.

Alternativas
Comentários
  • C 150,1 x C350,2 / C 500,3 = 0,44


ID
501871
Banca
CESPE / CEBRASPE
Órgão
Petrobras
Ano
2007
Provas
Disciplina
Estatística
Assuntos

Considere que a vazão V de um oleoduto seja uma variável aleatória que siga uma distribuição normal com média igual a 1.000 m3 por dia e desvio-padrão igual a 500 m3 por dia. Nessa situação, julgue os itens subseqüentes.

A probabilidade de V ser igual a 1.000 m3 por dia é superior a 0,01.

Alternativas
Comentários
  • P(X=c) onde c é uma constante, em uma distribuição, é por definição igual a zero

  • Em uma distribuição contínua a probabilidade está relacionada a um intervalo e não a um ponto específico.

    Ex: Qual a probabilidade de uma pessoa no Brasil possuir, EXATAMENTE, 1,8123 m de altura.

    P(X) = 0

    Agora se a pergunta fosse: Qual a probabilidade de pessoas no Brasil com altura entre 1,98 e 2,10 m?

    R: A probabilidade provavelmente é baixa, no entanto, ela existe.

  • Exatamente o que os colegas disseram acima, a probabilidade para variáveis contínuas de um numero exatamente é igual a zero, variáveis contínuas estão ligadas a intervalos, e para ajudar no raciocínio teremos;

    P(z) = (V-media)/Desviopadrao

    P(z) = (1000-1000)/500

    P(z) = 0/500 = 0


ID
513550
Banca
FMP Concursos
Órgão
TCE-RS
Ano
2011
Provas
Disciplina
Estatística
Assuntos

Dois professores corrigem a prova de redação de um concurso público. O professor A corrige o dobro de provas do que o professor B. Sabe-se que 60% das provas corrigidas pelo professor A tiveram nota superior a 7, enquanto apenas 20% das provas corrigidas pelo professor B tiveram nota superior a 7. Se um candidato teve conceito não superior a 7, a probabilidade de sua prova ter sido corrigida pelo professor A é:

Alternativas
Comentários
  • Acredito que a resolução seja:

    1) Professor A corrige o dobro de provas do que o professor B, logo:
    Supondo que A corrigiu 100 provas, então B corrigiu 50 provas.

    2) 60% das provas corrigidas pelo professor A tiveram nota superior a 7, enquanto apenas 20% das provas corrigidas pelo professor B tiveram nota superior a 7, logo:
    60% de 100 provas = 60 provas que o professor A corrigiu que tiveram nota superior a 7 e 40 provas com nota INFERIOR a 7.
    20% de 50 provas = 10 provas que o professor B corrigiu que tiveram nota superior a 7 e 40 provas com nota INFERIOR a 7.

    3) Se um candidato teve conceito não superior a 7, a probabilidade de sua prova ter sido corrigida pelo professor A é:
    Se o candidato tirou nota INFERIOR a 7, o universo é (40+40) = 80
    Logo a probabilidade de o professor A ter corrigido a prova deste candidato é:
    P (A) = 40/80 = 0,50

  • Essa é uma questão em que pode ser usado o Teorema da Bayes para resolve-la. Só lembrando, esse teorema é utilizado para calcular a probabilidade de um evento A acontecer, dado que o evento B tenha ocorrido.

    Então:

    P (A1 / B) =                     P (A1 ∩ B)                   
                      P (A1 ∩ B) + P (A2 ∩ B) ... P (An ∩ B)

    O professor A corrigiu o dobro de provas do professor B. Logo a probabilidade da prova ter sido corrigida por cada um dos professores é a seguinte:

    P(A) = 2/3 e P(B) = 1/3

    As provas corrigidas pelo professor A tiveram 60% notas superiores a 7 enquanto as provas corrigidas pelo professor B tiveram apenas 20% notas superiores a 7. Mas a questão exige a probabilidade de notas inferiores a 7. Logo, a probabilidade da nota ser inferior a 7 em cada professor é:

    P(A inferior a 7) = 4/10 e P(B inferior a 7) = 8/10

    Lembrando que para calcular P (A ∩ B)  = P(A) x P(B) para eventos independentes, basta utilizar a fórmula acima:


    P (A / X inferior a 7) =                  P (A ∩ A inferior a 7)                 
                                  P (A ∩ A inferior a 7) + P (B ∩ B inferior a 7)    


    P (A / X inferior a 7) =              (2/3) x (4/10)            =    8/30   =  = 1 = 0,5
                                ((2/3) x (4/10)) + ((1/3) x (8/10))    16/30      16    2
  • Eu faria assim:
    objetivo descobrir P(A|<7) = ?
    número de provas A = 2*nb
    número de provas B = nb
    número de provas total = 2nb + nb = 3nb
     
    P(A |<7) = P(A ∩<7) / P(<7) 
    P(A ∩<7) = 0,4*2nb/3nb =0,266667
    P(<7) = (0,4*2nb+0,8nb)/3nb = 0,5333
    P(A|<7) = P(A∩<7)/P(<7) = 0,5
  • Com devido respeito aos estatísticos, o jurista aqui não precisou usar todas essas fórmulas pra chegar ao resultado.

    De forma mais simples, criei números.

    A corrige o dobro de B. Suponhamos que foram corrigidas 30 redações. Logo, 20 para A e 10 para B.

    Dessas, 40% das redações de A estão com nota abaixo de 7; Logo, 8 redações

    E 80% das redações de B estão com nota abaixo de 7; Logo, 8 também.

    Como há o mesmo número de redações com notas abaixo de 7, probabilidade de 50% pra cada um.

    CORRETA D
  • É facil..

    Primeiro suponha um valor total de provas,pois como a questão traz somente a porcentagem é possivel fazer uma suposição para realizar a questão.

    Vamos supor que o total de provas seja 300 e que o professor A corrija 200 e o B 100,pois A corrige o dobro de B

    Então teremos:

    A : corrige 200 provas,destas 60%(120 provas) são acima de 7 e o restante ,40% (equivalente a 80 provas) são abaixo de7

    B: corrige 100 provas,destas 20% ( 20 provas) são acima de 7 e o restante,80% (80 provas) são abaixo de 7


    A questão pede a probabilidade de se retirar uma prova abaixo de 7 e esta ter sido corrigido pelo professor A

    então  é só pegar o total de provas abaixo de 7 corrigidas pelo professor A e dividir pelo total de provas abaixo de 7

    ou seja ,           O QUE EU QUERO/ TOTAL   ------------->    80/ 160 = 0,5

ID
513859
Banca
FMP Concursos
Órgão
TCE-RS
Ano
2011
Provas
Disciplina
Estatística
Assuntos

Segundo o controle de qualidade de uma empresa, a probabilidade do seu produto apresentar falha é de 0,10. Três pessoas compram o produto. A probabilidade de somente duas dessas pessoas terem comprado o produto com falha é:

Alternativas
Comentários
  • Deve-se pensar da seguinte forma:
    Para cada consumidor com falha temos a probalidade de 0,10, ou seja, três pessoas:

    0,1 x 0,1 x 0,9 ( a última não comprou com defeito, apenas duas) = 0,009, essa é a chance de cada um comprar com defeito, como entraram 3 pessoas na loja é preciso somar 0,009 + 0,009 + 0,009 = 0,027
  • Probabilidade


    P(B=2) = Combin( 3, 1 ) * 0,9^1 * 0,1^2

    P(B=2) = 3 * 0,9 * 0,01

    P(B=2) = 0,027

    Letra C


  • Probabilidades:

    P(Falha) = 0,1

    P(Não ter Falha) = 0,9


    0,1 * 0,1 * 0,9 = 0,009

    0,1 * 0,9 * 0,1 = 0,009

    0,9 * 0,1 * 0,1 = 0,009


    Somadas essas 3 probabilidades, P = 0,027.

  • Questao de probabilidade binomial


ID
542158
Banca
CESGRANRIO
Órgão
Transpetro
Ano
2011
Provas
Disciplina
Estatística
Assuntos

Duas empresas diferentes produzem a mesma quantidade de aparelhos celulares, ou seja, ao se comprar um aparelho celular, a probabilidade de ele ter sido produzido por qualquer uma delas é a mesma. Cada aparelho produzido pela fábrica A é defeituoso com probabilidade 1%, enquanto cada aparelho produzido pela fábrica B é defeituoso com probabilidade 5%. Suponha que você compre dois aparelhos celulares que foram produzidos na mesma fábrica. Se o primeiro aparelho foi verificado e é defeituoso, a probabilidade condicional de que o outro aparelho também seja defeituoso é

Alternativas
Comentários
  • 1º Passo (possibilidades total): a probabilidade de defeituoso A: 1/100 e a Probabilidade de defeituoso B: 5/100

    1/100 + 5/100 = 6/100 (quando for "um" ou "outro", vc soma,mas, quando for "um" e "outro", vc multiplica)

     

    2º Restrição (Possibilidade desejada): Se o 1º é defeituoso logo o 2º também é:

    Probabilidade do 1º defeituoso A: 1/100

    Probabilidade do 2º defeituoso B: 1/100

    A probabilidade de "1º e o 2º" serem defeituoso é:1/100 x 1/100 = 1/10000

     

    Probabilidade do primeiro defeituoso B: 5/100. A probab. do segundo defeituoso: 5/100. A probabilidade de o primeiro "e o" segundo serem defeituoso: 5/100 x 5/100 = 25/10000

    (quando for "um" ou "outro", vc soma) = 25/10000 + 1/10000 = 26/10000

     

    3º Passo: Possibilidades desejadas dividida pela Possibilidade total:

    26/10000 / 6/100 = na divisão vc inverte a segunda fração e multiplica=  26/10000 x 100/6 = 2600/60000 = 13/300

     

    RESPOSTA: C

     

     


ID
542947
Banca
FCC
Órgão
TRT - 23ª REGIÃO (MT)
Ano
2011
Provas
Disciplina
Estatística
Assuntos

Placas de um circuito integrado são expedidas em lotes de 10 unidades. Antes de um lote ser aprovado um procedimento de controle de qualidade escolhe aleatoriamente e sem reposição 4 placas do lote. Se uma ou mais forem defeituosas, todo o lote é inspecionado. Supondo que num lote haja duas placas defeituosas, a probabilidade de que o controle de qualidade indique uma inspeção de todo o lote é

Alternativas
Comentários
  • Vamos lá:

    Se com pelo menos 1 defeituosa, o lote é inspecionado... vamos somar as probabilidades de encontrar 1.. e depois de encontrar 2 defeituosas.

    P1)  

    2/10 . 8/9 . 7/8 . 6/7 . P43 = 48/90

    P2)

    2/10 . 1/9 . 8/8 . 7/7 .  P42;2 = 12/90

    48/90 + 12/90

    60/90

    2/3

    Se ficou alguma dúvida >>> sergio.harger@gmail.com

    Abs,

    SH.
  • Tbm pode ser feito da seguinte maneira:

    Probabilidade de escolher 4 peças sem nenhum defeito:

    (8/10) * (7/9) * (6/8) * ( 5/7) = 1/3

    Logo, a probabildiade de escolher pelo menos uma peça com defeito é:

    1 - 1/3 = 2/3
  • Uma terceira solução seria:

     Distribuição hipergeométrica:

    Prob (de nenhuma com defeito) = (2 0)*(8 4) / (10 4) = 1/3

    Só que queremos a probabilidade de ter ao menos uma com defeito = 1 - 1/3 = 2/3 (letra B)

    amostra n = 4

    numerador (possibilidades de interesse)

    (2 0) = combinacao de 2, 0 a 0 = eu pego 0 bolas com defeito dentre 2 com defeito
    (8 4) = eu pego 4 bolas normais dentre 8 normais

    denominador (total de possibilidades)

    (10 4) = pegar 4 bolas dentre o total de 10



     


ID
542953
Banca
FCC
Órgão
TRT - 23ª REGIÃO (MT)
Ano
2011
Provas
Disciplina
Estatística
Assuntos

Três máquinas: A, B e C de uma determinada indústria produzem a totalidade das peças de certo tipo que são utilizadas na fabricação de um motor de um automóvel. Sabe-se que a A e B produzem cada uma 30% das peças e C produz 40%. Sabe-se que 5%, 10% e 2%, respectivamente, das produções de A, B e C são defeituosas. Uma peça é selecionada, aleatoriamente, da produção conjunta das três máquinas. A probabilidade de ela ter sido fabricada por A, sabendo-se que é defeituosa, é

Alternativas
Comentários
  • Queria saber se tem a resolução.

    obrigado.

    helio.
  • Probabilidade total de peças defeituosas:

    De A = 0,3 * 0,05
    De B = 0,3 * 0,10
    De C = 0,4 * 0,02

    Probabilidade de peças defeituosas = 0,053 = Somas das  equaçoes anteriores

    Probabilidade de A ser defeituoso = 0,3 * 0.05 = 0,015

    A pergunta é : Probabilidade de A ser defeituoso /   Probabilidade de peças defeituosas = 0,015 / 0,053 = 15/53

ID
542959
Banca
FCC
Órgão
TRT - 23ª REGIÃO (MT)
Ano
2011
Provas
Disciplina
Estatística
Assuntos

Um procedimento de controle de qualidade foi planejado para garantir uma proporção máxima (p) de 10% de itens defeituosos na produção. A cada uma hora sorteia-se uma amostra de 4 peças da produção e havendo mais do que 1 peça defeituosa nesta amostra a produção é parada para verificação. A probabilidade de se parar a produção desnecessariamente, quando p = 0,10, está dentro do intervalo

Alternativas
Comentários
  • Probabilidade de nenhum item defeituoso na amostra = 0,9^4

    Probabilidade de exatamente 1 item defeituoso na amostra = C4,1 * ( 0,9^3) * 0,1

    Soma das probabilidades anteriores = 0,9477

    A probabildiade que é perguntada da questão é : 1 - 0,9477 = 0,0523, alternativa C

    Isso é assim pq ele perguta a probabilidade de se parar a produção, que ocorre quando mais de 1 item é defeituoso!!!!
  • Questão anulável. Em nenhum momento, foi dito que a seleção era com reposição.
  • Além do problema nao ter explicitado que é com reposicao. Pecou por deixar o entendimento nas entrelinhas: em uma produção depreende-se que a amostragem é sem reposicao. Não faz sentido voce avaliar se um item tem ou nao defeito e voltar com ele para amostra.

    Imagina uma amostra com 100 objetos, dentre os quais, 99 sao perfeitos, e somente 1 tenha defeito. Sao feitas 2 averiguacoes (com reposicao), e acontece de nessas 2 averiguacoes, pegarmos justamente o objeto defeituoso. Aí concluiríamos que o lote está 100% comprometido, quando na verdade, só há uma peça com defeito dentre 100. Portanto, não faz sentido que seja uma amostragem com reposição. O que se pode inferir das entrelinhas é que trata-se de uma amostragem sem reposição:

    Mais um motivo para questão ter sido anulada.


ID
548839
Banca
CESGRANRIO
Órgão
Petrobras
Ano
2011
Provas
Disciplina
Estatística
Assuntos

Uma fábrica está investindo na produção de uma nova bomba altamente sofisticada. Uma pesquisa que considera as duas maiores empresas da região mostrou que a probabilidade da empresa 1 comprar tal bomba é de 0,7, e a da empresa 2 é de 0,5. Já a probabilidade de a compra ser efetuada pelas duas empresas corresponde a 0,4. Nessa perspectiva, a probabilidade de a bomba ser comprada por pelo menos uma das empresas é

Alternativas
Comentários
  • ..raciocinei da seguinte forma:

    empresa A tem 70/100 de comprar a bomba

    empresa B tem 50/100 de comprar a bomba

    empresa A e B tem 40/100 de comprar a bomba

    P = P(A) + P(B) - P(AB) =  80 =  0,8

                     100                100

    Resposta item D


ID
548842
Banca
CESGRANRIO
Órgão
Petrobras
Ano
2011
Provas
Disciplina
Estatística
Assuntos

No estoque de uma empresa, há trinta compressores do mesmo tipo. Seis deles, no entanto, são defeituosos. Um funcionário seleciona aleatoriamente dois desses
compressores. Considerando-se que, uma vez selecionados, não há reposição de qualquer dos equipamentos, qual a probabilidade de ambos serem defeituosos?

Alternativas
Comentários
  • Probabilidade de dois compressores com defeito:
     6 x 5 = 30

    Probabilidade Total de escolha dos compressores:

    30 x 29  

    Probabilidade de dois compressores com defeito com relação ao total:

    30 / ( 30 X 29 ) = 1/29

  • Primeira escolha -> chances 6/30

    Segunda escolha -> chances 5/29

    6/30 x 5/29 = 1/29


ID
554830
Banca
FCC
Órgão
INFRAERO
Ano
2011
Provas
Disciplina
Estatística
Assuntos

Um dado é viciado de tal modo que a probabilidade de ocorrer face par é duas vezes mais provável do que ocorrer face ímpar. O dado é lançado duas vezes independentemente. Considere os seguintes eventos:

A = a soma dos pontos das faces é 6;
B = o número da face do primeiro dado é menor do que 3.

Nessas condições, a probabilidade de A, sabendo que ocorreu B, é

Alternativas
Comentários
  • Se o dado possui a característica de vicio, é só entender quantos são os resultados possíveis e quantos são os resultados desejados dentro das características apresentadas.
    Temos então 9 resultados possíveis: 1, 2, 2, 3, 4, 4, 5, 6, 6 (os pares tem probabilidade duas vezes maior de acontecer)

    1º DADO,NUMEROS MENORES QUE 3 TEMOS : 1, 2, 2 (três resultados possíveis)

    2º DADO,OS NUMEROS POSSIVEIS PARA FECHAR A SOMA 6,TEMOS: 5, 4, 4.
    Temos então cada 4 podendo fechar com qualquer 2 (quatro possibilidades) e o 5 fechando com o 1. Total de 5 possibilidades.

    Então no primeiro dado temos 3 possibilidades em 9
    e no segundo dado: 5 em 9

    3/9 * 5/9 = 5/27!
  • P(1)=P(3)=P(5)=1/9
    P(2)=P(4)=P(6)=2/9
    Dois lançamentos
    A = Soma 6
    B = 1º lçto = 1 ou 2

    Possíveis somas 6
    P(3+3)=1/9x1/9=1/81
    P(2+4)=2/9x2/9=4/81
    P(4+2)=2/9x2/9=4/81
    P(1+5)=1/9x1/9=1/81
    P(5+1)=1/9x1/9=1/81
    Total =11/81= P(A)

    P(AB)=P(1+5)+P(2+4)=1/81+4/81=5/81

    P(B)=P(1ºlçto=1)+P(1ºlçto=2)=1/9+2/9=3/9=1/3

    Probabilidade Condicional: P(A/B)=P(AB)/P(B)

    P(A/B)=(5/81)/(1/3)=5/81x3=15/81=5/27

  • P(B) = (1/9)+(2/9) = 1/3 => 1/9 possibilidade de ser número 1 e 2/9 possibilidade de ser o número 2.

    P(A e B) = (1/9).(1/9) + (2/9).(2/9) = 5/81 => os primeiros termos multiplicando representam a possibilidade de a soma ser 6 com dois números ímpares (1+5) e a segunda dois números pares (2+4)

    P(A/B) = P(A e B)/P(B) = (5/81)/(1/3) = 5/27




ID
554848
Banca
FCC
Órgão
INFRAERO
Ano
2011
Provas
Disciplina
Estatística
Assuntos

Atenção: Para resolver às questões de números 52 e 53, considere os dados abaixo:

A empresa de aviação T tem 4 balcões de atendimento ao público: A, B, C e D. Sabe-se que, num determinado dia, os balcões A e B atenderam, cada um, a 20%; C e D atenderam, cada um, a 30% do público que procurou atendimento em T. Sabe-se ainda que A, B, C e D atenderam, respectivamente, 5%, 15%, 10% e 20% de pessoas com atendimento prioritário (idosos, deficientes, gestantes ou mães com crianças no colo, etc).


Selecionando-se ao acaso uma pessoa atendida por T, nesse mesmo dia, a probabilidade dela ter sido atendida no balcão C, sabendo-se que era do grupo de atendimento prioritário, é igual a

Alternativas
Comentários
  • Se naquele dia a cia T atendeu 100 pessoas nos balcões A, B, C e D, o atendimento foi de respectivamente 20, 20, 30 e 30 pessoas. Se o balcão A atendeu 5% de pessoas com atendimento prioritário, foi atendida 1 pessoa, pois 20 x 0,05 = 1. Seguindo o mesmo raciocínio, o balcão B atendeu 3 pessoas (20 x 0,15), o balcão C atendeu 3 pessoas (30 x 0,10) e o balcão D atendeu 6 pessoas (30 x 0,20) totalizando 13 pessoas com prioridade no atendimento.
    Portanto o balcão C atendeu 3 de 13 pessoas com prioridade. Resposta alternativa E.

ID
554851
Banca
FCC
Órgão
INFRAERO
Ano
2011
Provas
Disciplina
Estatística
Assuntos

Atenção: Para resolver às questões de números 52 e 53, considere os dados abaixo:

A empresa de aviação T tem 4 balcões de atendimento ao público: A, B, C e D. Sabe-se que, num determinado dia, os balcões A e B atenderam, cada um, a 20%; C e D atenderam, cada um, a 30% do público que procurou atendimento em T. Sabe-se ainda que A, B, C e D atenderam, respectivamente, 5%, 15%, 10% e 20% de pessoas com atendimento prioritário (idosos, deficientes, gestantes ou mães com crianças no colo, etc).


Selecionando-se ao acaso e com reposição cinco pessoas atendidas no balcão D, nesse mesmo dia, a probabilidade de exatamente duas terem sido do grupo de atendimento prioritário é de

Alternativas
Comentários
  • Se naquele dia  a Cia T atendeu 100 pessoas, o balcão D atendeu 30% deles, ou seja, 30 pessoas. Dessas 30 pessoas, 20% eram de atendimento prioritário, ou seja, 30 x 0,20 = 6 pessoas. A probabilidade de uma pessoa atendida no balcão D ser de atendimento prioritário é de 6/30 = 1/5 ou 20%. Consequentemente a probabilidade de uma pessoa atendida nesse balcão não ser de atendimento prioritário é de (1,00 - 0.20) = 0,80 ou 80%. Queremos saber qual a probabilidade de, em uma seleção de cinco pessoas atendidas por esse balcão, exatamente duas serem do grupo de atendimento prioritário. Essa probabilidade é dada pela multiplicação das probabilidades de dois serem de atendimento prioritário e tres serem de atendimento não prioritário, ou seja, P = 0,20 x 0,20 x 0,80 x 0.80 x 0.80 = 0, 02048. No entanto esse evento ocorre dez vezes, a ordem não importando podemos utilizar a fórmula da combinação, ou seja, C n, k = n!/((n-k)! * k!) = 5!/((5 - 2)!*2!) = 10. Então retomando, 10 x P = 10 x 0,02048 = 0,2048, alternativa B
  • Trata-se de uma questão de Distribuição Binomial. Para identificarmos esse tipo de questão, devemos verificar a ocorrência de 3 fatores, quais sejam:
    i) Dualidade - É / Não é do grupo de atendimento prioritário;
    ii) Reposição - A questão diz ser e;
    iii) Ordem não explicitada na questão.
    Assim, teremos (5 2) x 0,2 x 0,2 x 0,8 x 0,8 x 0,8, onde 0,2 é a probabiidade de ser de atendimento prioritário e 0,8 de não ser.
    Então (5x4)/(2x1) x 0,0248 = 10x0,0248 = 0,248.
    Gabarito: B 
    Bons estudos!
  • Vamos supor que o total sejam 100 pessoas  e que o balcão D tenha atendido 30 pessoas(30%) e dessas 30 pessoas 6 (20%) tiveram atendimento prioritario

    - como não há equidade de possibilidades entre as pessoas,pois a possibilidade maior de sair uma pessoa sem atendimento especial é maior que as que receberam atendimento especial,então primeiramente temos que encontrar o total de possibilidades...Primeiro eu encontro o total e depois encontro o que eu quero desse total

    -então selecionando 5 pessoas das 30 temos  30 x 30 x 30 x 30 x 30 =  24.300.000

    Como há reposição ,então para a primeira selecionada eu terei 30 pessoas,para a segunda terei 30 novamente ,para a terceira 30 e assim sucessivamente

    Assim encontramos a probabilidade de cada uma das pessoas ser selecionada que é 24.3000 

    Como a questão pede exatamente que 2 pessoas selecionadas tenham sido do atendimento prioritario então teremos:

     6 x 6 x24 x 24 x 24 =497.664  ( observe que não foi multiplicado por 30 pois a questão exige somente 2 pessoas de atendimento prioritario,então eu fiz 30 que é o total menos 6 que receberam o atendimento prioritario)


    Agora é só dividir o que eu quero pelo total =                      497.664/ 24.300.000 = 0,02048



ID
554854
Banca
FCC
Órgão
INFRAERO
Ano
2011
Provas
Disciplina
Estatística
Assuntos

Um experimento pode resultar em “sucesso” ou “fracasso” com probabilidades 0,25 e 0,75, respectivamente. Considere a variável aleatória X= número de “fracassos” antes de ocorrer o primeiro “sucesso”. A probabilidade de X ser pelo menos 2 e a média de X são dadas, respectivamente, por

Alternativas
Comentários
  • probabilidade de x ser ao menos 2 = 1 - ( probabilidade de x ser 0 ou 1 ) = 1 - 0,4375 = 0,5625

    uma vez que:

    prob(x=0) = 0,25
    prob(x = 1) = 0,25*0,75

    as duas ultimas probabilidades somadas: 0,4375

    Média de x: A banca se equivocou ao dizer que a média de x é 4. Na verdade a média de x é 3. Pois, x é definido como o número de fracassos ANTES do primeiro sucesso, ou seja, nao inclui o ensaio sucesso no número de tentativas. Sendo assim, o número de fracassos em média para se ter o primeiro sucesso é 3. Em outras palavras: a cada 4 ensaios tem-se em média um sucesso, haja visto que prob de sucesso = 1/4.

    x não é o número de ensaios para se ter o primeiro sucesso, e sim o numero de fracassos antes do primeiro sucesso. Média de x = 3

    Portanto, essa questao deveria ser anulada.


  • Trata-se da Distribuição Geometrica:

    Definida por P[X=J] = (1-P)^J (P)

    A experança de X deste modelo ou seja numero de fracassos esperados até que ocorra o primeiro sucesso é dado por:

    E(X)= 1(1-P) / P

    ou seja 0,75/0,25 = 3

    Concordo que a questão deveria ser anulada.

     

  • Fórmulas da Distribuição Geométrica:

    Probabilidade: P(X=n) = (1-p)^n-1 x p

    Média/Valor Esperado: E(x) = 1/p

    Variância: V(x) = (1-p) / p^2

    Vamos à questão: ela quer no mínimo 2 Fracassos antes do 1º Sucesso, logo: F/F/S no mínimo.

    Sendo assim, o que eu quero é 100% - o que não quero: Q = 1 - ÑQ

    Quero Sucesso na 3ª tentativa, no mínimo, logo:

    P(X=1) = 0,75^1-1 x 0,25 = 0,25

    P(X=2) = 0,75^2-1 x 0,25 = 0,1875

    Dessa forma, a probabilidade de X maior ou igual a 3 é: P = 1 - 0,25 - 0,1875 = 0,5625

    A média é simplesmente 1/p, logo: 1 / 0,25 = 4.


ID
556207
Banca
CESGRANRIO
Órgão
EPE
Ano
2010
Provas
Disciplina
Estatística
Assuntos

Lâmpadas foram classificadas em 3 grupos, dependendo do tempo de durabilidade. As lâmpadas classificadas como de curta duração são aquelas em que o tempo de vida é inferior a 500 horas; as classificadas como de média duração têm tempo de vida com mais de 500 e menos de 800 horas e as demais têm longa duração.
Experiências anteriores estimam que as probabilidades de as lâmpadas serem classificadas como de curta, média e longa duração são, respectivamente, 0,5, 0,3 e 0,2.
Selecionando-se n lâmpadas, a probabilidade de haver a lâmpadas de curta duração, b lâmpadas de média duração e c lâmpadas de longa duração, sendo a + b + c = n e a > 0; b > 0 e c > 0, é

Alternativas
Comentários
  • Distribuição polinomial ou multinominal


ID
563182
Banca
CESGRANRIO
Órgão
Petrobras
Ano
2010
Provas
Disciplina
Estatística
Assuntos

Uma empresa de pequeno porte possui 10 funcionários. Um levantamento socioeconômico indicou que 5 funcionários residem em residência própria. Se for escolhida aleatoriamente uma amostra de 4 funcionários, qual a probabilidade de que 3 funcionários residam em casa própria ?

Alternativas
Comentários
  • Combinações possíveis (Própria / Alugada):

    PPPA = 5/10.4/9.3.8.5/7 +
    PPAP = 5/10.4/9.5/8.3/7 +
    PAPP = 5/10.5/9.4/8.3/7 +
    APPP = 5/10.5/9.4/8.3/7

    Finalmente: 4 x [5 / (21 . 4)] = 5 / 21= 0,2380952380952381

  • Distribuição hipergeométrica:

    notação de combinação C(n,p) 

     

    P(sucesso=3) = [ C(5,3) * C(5,1) ] / C(10,4)

    P(sucesso=3) = 0,238


ID
563290
Banca
CESGRANRIO
Órgão
Petrobras
Ano
2010
Provas
Disciplina
Estatística
Assuntos

As técnicas de simulação são muito importantes em uma grande variedade de projetos quando estes apresentam cálculos muito complexos ou experimentos reais muito dispendiosos. Na base da simulação, tem-se a necessidade de geração de números pseudoaleatórios, quando as duas principais preocupações são: (1) um possível número deve ter a mesma probabilidade de ocorrer que qualquer outro dentre os demais possíveis números e (2) deve existir independência entre as ocorrências, isto é, a probabilidade de ocorrência de um número não deve ser afetada pelas eventuais ocorrências dos demais possíveis números. Os métodos de geração mais adotados na prática são: congruência mista (mixed congruential method), congruência multiplicativa (multiplicative congruential method) e congruência aditiva (additive congruential method). Considere os números inteiros K, L, M e N, tais que: 0 < K < M; 0 < L < M e N = 1, 2, 3... Para serem gerados números pseudoaleatórios entre 0 e M-1, inicia- se com uma semente X0 aleatoriamente escolhida e adota-se a relação de recorrência XN+1 = f(XN, XN-1, K,L)(módulo M), isto é, XN+1 é o resto da divisão de f(XN, XN-1,K,L) por M. Nessas condições, quando

Alternativas

ID
563296
Banca
CESGRANRIO
Órgão
Petrobras
Ano
2010
Provas
Disciplina
Estatística
Assuntos

Na simulação da operação de uma planta industrial, supõe-se que ela pode apresentar dois estados: ou operou normalmente ou operou com alguma anomalia. Se um dia operou normalmente, a probabilidade de apresentar alguma anomalia no dia seguinte é 70%. Quando um dia operou com alguma anomalia, a probabilidade de operar normalmente no dia seguinte é 60%. Independente de como esteja operando atualmente, após muitos dias de operação, a probabilidade de concluir um dia operando normalmente é de, aproximadamente,

Alternativas
Comentários
  • No longínquo período de operação, a probabilidade de um dia operar OK é x. De não operar OK é y.

    x + y = 1

    A partir dai conclui-se que 0,3*x + 0,6*y = x -> 7x = 6y

    6x + 6y = 6

    13x = 6

    x = 6/13 = 0,01*(600/13) = 0,01*46 = 46%


ID
563299
Banca
CESGRANRIO
Órgão
Petrobras
Ano
2010
Provas
Disciplina
Estatística
Assuntos

Com base em dados históricos, verifica-se que, se uma linha de produção apresenta um índice de falhas inferior a 5% em determinado dia, a probabilidade de operar com mesmo nível de qualidade no dia seguinte é de 80%. Por outro lado, se opera com índice de falhas igual ou superior a 5% em algum dia, a probabilidade de voltar a operar com índice inferior a 5% no dia seguinte é de, apenas, 30%. Se, na simulação desse processo, verifica-se que a probabilidade de estar operando com índice de falhas inferior a 5% em algum dia é de 70%, a probabilidade de assim estar operando dois dias depois é de

Alternativas

ID
563371
Banca
CESGRANRIO
Órgão
Petrobras
Ano
2010
Provas
Disciplina
Estatística
Assuntos

Uma moeda honesta foi lançada 4 vezes. Uma pessoa contou o número de caras nos três primeiros lançamentos e outra contou o número de caras nos três últimos lançamentos. A probabilidade de que ambas tenham contado exatamente duas caras é

Alternativas
Comentários
  • K-Cara; C-Coroa
    K C K K = 1/16
    C K K C = 1/16

    K K C K = 1/16


    1/16 *3 = 3/16

  • Eu inerpretei que cada pessoa tinha somado 2, e não que a soma das duas tenha dado 2. 

  • Mas é isso mesmo, Luciana. Cada uma das pessoas vai verificar duas caras individualmente conforme o enunciado e cálculo acima.

    Se fosse a soma, seria só possível a combinação KCCK, onde cada um veria uma cara e a soma daria 2 caras e a resposta seria 1/16.


ID
563374
Banca
CESGRANRIO
Órgão
Petrobras
Ano
2010
Provas
Disciplina
Estatística
Assuntos

Uma prova consta de 35 questões do tipo múltipla escolha, com 5 opções cada uma, onde apenas uma opção é verdadeira. Um candidato que não sabe resolver nenhu- ma das questões vai respondê-las aleatoriamente. Ele sabe que as respostas certas das 35 questões estão distribuídas igualmente entre as opções A,B,C,D e E, e resolve marcar suas respostas seguindo esse critério: escolherá aleatoriamente 7 questões para marcar a opção A, outras 7 para a B, e assim sucessivamente. A probabilidade de ele acertar todas as questões é

Alternativas
Comentários
  • Letra A

     
    (7/35*6/34*5/33*4/32*3/31*2/30*1/29) = 7 ! / 35 x ... x 29



     Letra B


    (7/28*6/27*5/26*4/25*3/24*2/23*1/22) = 7 ! / 38 x ... x 22



    Letra C


    (7/21*6/20*5/19*4/18*3/17*2/16*1/15) = 7 ! / 21 x ... x 15

     


     Letra D


    (7/14*6/13*5/12*4/11*3/10*2/9*1/8)  = 7 ! / 14 x ... x 8



    Letra E


    (7/7*6/6*5/5*4/4*3/3*2/2*1/1)  = 7 ! / 7 x ... x 1



    ( 7 ! / 35 x ... x 29 ) x ( 7 ! / 38 x ... x 22 ) x (  7 ! / 21 x ... x 15 ) x ( 7 ! / 14 x ... x 8 ) x ( 7 ! / 7 x ... x 1 ) = 7 ! ^5 / 35 !

  • Total de possibilidades: P(7,7,7,7,7);35 = 35!/(7!)^5

    Casos favoráveis: 1  

    Probabilidade: 1/[35!/(7!)^5)] = (7!)^5/35! 


ID
563977
Banca
CESGRANRIO
Órgão
Petrobras
Ano
2010
Provas
Disciplina
Estatística
Assuntos

Dois eventos de um espaço amostral são independentes quando

Alternativas

ID
563995
Banca
CESGRANRIO
Órgão
Petrobras
Ano
2010
Provas
Disciplina
Estatística
Assuntos

Quando se lança uma certa moeda, a probabilidade de o resultado ser cara é p. A moeda foi lançada dez vezes, sucessivas e independentes, e o resultado foi de 2 caras e 8 coroas. Tendo em vista este experimento, a estimativa de máxima verossimilhança de p é

Alternativas

ID
566098
Banca
CESGRANRIO
Órgão
Petrobras
Ano
2010
Provas
Disciplina
Estatística
Assuntos

Um dado é lançado duas vezes e todos os seis resultados possíveis para cada lançamento são equiprováveis. A probabilidade condicional para que ambos sejam pares quando pelo menos um dos resultados destes dois lançamentos for um número par será igual a

Alternativas

ID
569914
Banca
FCC
Órgão
BACEN
Ano
2006
Provas
Disciplina
Estatística
Assuntos

A probabilidade de um associado de um clube pagar sua mensalidade com atraso é de 5%. Entre 5 associados escolhidos aleatoriamente, a probabilidade de pelo menos um pagar sua mensalidade sem atraso é

Alternativas
Comentários
  • O gabarito está correto. Ele pede o probabilidade de pelo menos um pagar SEM atraso.

    Isso é equivalente a: 1 - probabilidade de todos atrasarem

    Prob todos atrasarem = (0,05)5

    Resposta: 1- (0,05)5

    LETRA E
  • LETRA E.

    a condição dada pelo problema é que pelo menos alguém não atrase. Certo. A conta fica mais simples, então, se calcularmos a probalidade de todos atrasarem e diminuir esse valor de 1 para encontrar a probabilidade.

    Ou seja, a probabilidade de ningúem pagar em dia é:

    0,05 x 0,05 x 0,05 x 0,05 x 0,05 = 0,05 ^ 5 

    1- 0,05 ^5 - Alternativa E

ID
573859
Banca
CESGRANRIO
Órgão
Petrobras
Ano
2010
Provas
Disciplina
Estatística
Assuntos

Um sorteio é realizado com duas urnas, I e II. As urnas são escolhidas ao acaso. A urna I contém 2 bolas brancas e 6 pretas. A urna II contém 4 bolas brancas e 4 pretas. Se a bola sorteada for branca, qual a probabilidade de ter sido da urna I?

Alternativas
Comentários
  • Em sendo a bola sorteada, uma bola branca, as experiências que levam à bolas pretas devem ser ignoradas. 

    1. Casos possíveis de retiradas de bolas brancas = 2 + 4 = 6 

    2. Casos favoráveis para a bola branca ter vindo da urna I = 2 

    3. Probabilidade = P = 2/6 = 1/3 

    Resposta:  P = 1/3  opção (A) 

  • P( urna 1/ bola Branc) = P (urna 1 e bola branca) / P (bola branca) = 2/8   /  (2/8+4/8) = 1/3

     

    letra A 

     

    bons estudos !


ID
583489
Banca
CESGRANRIO
Órgão
Petrobras
Ano
2006
Provas
Disciplina
Estatística
Assuntos

O Teorema de Bayes é uma relação que expressa uma probabilidade:

Alternativas
Comentários
  • Teorema de Bayes: 

    P(A/B)=P(B/A)P(A) / P(B)

    P(A/B) e P(B/A) : probabilidades condicionais

    P(A) e P(B) : probabilidades marginais

    .: condicional, em termos de outras probabilidades condicionais e marginais (Letra a)


ID
600439
Banca
CESGRANRIO
Órgão
BNDES
Ano
2011
Provas
Disciplina
Estatística
Assuntos

Em uma urna, são colocadas 2 bolas brancas e 4 pretas. Alberto e Beatriz retiram bolas da urna alternadamente, iniciando-se com Alberto, até que a urna esteja vazia. A probabilidade de que a primeira bola branca saia para Alberto é

Alternativas
Comentários
  • Podemos identificar 3 situações possíveis:

    Situação 1: Alberto tirar branca na primeira <=> P = 2/6 = 1/3

    OU

    Situação 2: Alberto tirar preta E Beatriz tirar preta E Alberto tirar branca <=>

    P = 4/6 x 3/5 x 2/4 = 24/120 = 1/5

    OU

    Situação 3: Alberto tirar preta E Beatriz tirar preta E Alberto tirar preta E Beatriz tirar pretaE Alberto tirar branca <=> P = 4/6 x 3/5 x 2/4 x 1/3 x 2/2 = 48/720 = 1/15

     

    então,

    P = 1/3 + 1/5 + 1/15 = 3/5

    OBS: o OU implica na utilização de soma e o implica em multiplicação.

  • Outra froma de resolver é calcular o total de chances de não sair a branca e subtrair das chances do Alberto.

    Considerando somente as bolas pretas: (A)4/6 x (B)3/5 x (A)2/4 x (B)1/3 = 1/15 (já simplificado). 

    Quantas chances Alberto vai ter de tirar a bola branca? 3 em um total de 6. Considerando que só temos 2 bolas brancas, então, 2/3.

    Resolução: 1/15 - 2/3 = 9/15 = 3/5


ID
636343
Banca
FGV
Órgão
Senado Federal
Ano
2008
Provas
Disciplina
Estatística
Assuntos

Avalie as afirmativas a seguir acerca de estatísticas suficientes minimais:
I. Uma estatística é suficiente minimal se é suficiente e se é uma função de alguma outra estatística suficiente.
II. Se um estimador de máxima verossimilhança é uma estatística suficiente então ele é uma estatística suficiente minimal.
III. Se um estimador de Bayes é uma estatística suficiente, então ele é uma estatística suficiente minimal.
Assinale:

Alternativas

ID
636373
Banca
FGV
Órgão
Senado Federal
Ano
2008
Provas
Disciplina
Estatística
Assuntos

Um estatístico de uma companhia telefônica deseja estimar a proporção p de clientes satisfeitos com a introdução de um novo tipo de serviço. Suponha que o número de clientes da companhia seja grande. Sabe-se, com base em experiências anteriores, que p deve estar próxima de 0,50. O menor tamanho de amostra que ele deve considerar de modo a garantir com probabilidade de 95% um erro absoluto de estimação de no máximo 0,02 é:

Alternativas
Comentários
  • Uma proporção de uma população pode ser estimada com um intervalo de confiança da seguinte forma:

    p* - Zi x Raiz( p*(1-p*)/n ) < p < p* + Zi x Raiz( p*(1-p*)/n )

    p* é a estimativa conhecida pela experiência: p* = 0.5
    Zi é obtido da tabela normal para intervalo de confiança 95%: Zi = 1.96

    Logo,

    Zi x Raiz( (p*(1 - p*)/n ) = E = 0.02

    Resolva para n e encontrará 2401.

  • Trata-se de um problema que aborda o método de amostragem para Proporção Populacional sem fator de correção (viste que a população é considerada infinita)

    Intervalo de Confiança para Uma Proporção Populacional
    Um intervalo de confiança para uma proporção populacional é dado por:
     

    onde:
         é a proporção amostral
       é o erro padrão da proporção amostral e é dado por:
     
     
     
    O intervalo de confiança é construído por:


    Como o exercício pediu o n desta amostragem, temos:

      A fórmula para determinar o tamanho amostral no caso de estimativa de proporções é:
         onde
     p  =    é a proporção estimada, baseada na experiência passada ou em uma amostra piloto
     Z  =  é o valor da variável normal padrão associado ao grau de confiança adotado.
     = é o máximo erro permissível que o pesquisador tolera.

    n = 0,5 *(1-0,5) * (1,96/0,02)^2 = 2041 amostras

ID
641899
Banca
FCC
Órgão
TCE-PR
Ano
2011
Provas
Disciplina
Estatística
Assuntos

Uma urna contém 3 bolas brancas, 4 pretas e 3 amarelas. Desta urna, três bolas são selecionadas ao acaso e com reposição. A probabilidade de que, entre as 3 selecionadas, no máximo duas sejam pretas é

Alternativas
Comentários
  • - Probabilidade de que as três sejam pretas: 4/10 . 4/ 10 . 4/ 10 = 64 / 1000

    Obs.: as probabilidades se repetem pois há reposição

    Como quero no máximo que 2 sejam pretas eu faço 1 - 0,064 = 0,936

    RESPOSTA B
  • Obrigado Igor por comentar, porém, não ficou muito claro a resolução dessa questão.
    Poderemos resolvê-la da seguinte forma:

    Seja X, nossa variável aleatória definida como sendo "quantidade de bolas pretas serem selecionadas".
    Neste caso, nossa variável aleatória assumiria os seguintes valores:  0, 1, 2, 3.
    Então, como o interesse está em ter no máximo duas bolas  pretas, então, teríamos o seguinte:

    P(X<=2) = P(X=0) + P(X=1) + P(X=2)     (expressão 1)

    Nossa variável aleatória X, segue distribuição binomial com a seguinte função de probabilidade:

    P(X = x) = [ n! / x!(n-x)! ] . px (1-p)n-x        (expressão 2)

    em que,
    X maiúsculo é a nossa variável aleatória;
    x minúsculo é uma realização da variável aleatória;
    n tamanho da amostra (número de retiradas da urna);
    !  símbolo que representa o fatorial;
    p é a probabilidade de sucesso;
    (1-p) é a probabilidade de fracasso.

    Vamos lá:

    Para facilitar nossa notação, chamaremos as bolas de: B(Branca) , P(Preta) e A(Amarela) .
    De acordo com a questão, temos as seguintes probabilidades:

    P(B) = 3/10 = 0,3
    P(P) = 4/10 = 0,4 = p (nossa probailidade de sucesso)
    P(A) = 3/10 = 0,3

    Lembrando que o valor "10" é a quatidade de bolas na urna (tamanho da população).
    Então, de acordo com a expressão 2 teremos o seguinte:

    P(X = 0) = [ 3! / 0!(3-0)! ] . 0,40 (1-0,4)3-0 = 0,216
    P(X = 1) = [ 3! / 1!(3-1)! ] . 0,41 (1-0,4)3-1 = 0,432
    P(X = 2) = [ 3! / 2!(3-2)! ] . 0,42 (1-0,4)3-2 = 0,288

    E agora, voltando na expressão 1, teremos o resultado final:

    P(X<=2) = 0,216 + 0,432 + 0,288 = 0,936

    espero ter ajudado e um abraço a todos!

    Att.
    André Barbosa V. da Silva
     



  • Resposta Correta: Letra B
     Caros colegas, quero parabenizá-los por ambos comentários postados!
    Igor resolveu o problema pela diferença entre a totalidade da probabilidade menos o evento que não é favorável, ou seja:
    P (x≤2) = 1 - P (x = 3)
    P(X≤2) = P(X=0) + P(X=1) + P(X=2) + P(X=3) - P(X=3)
    ΣP(X≤Xi) = 1
    de i até n
    Todos eventos favoráveis - o evento não favorável em um espaço amostral possível
     Meu Xará André, resolveu o problema pelo método da Distribuição Binomial.... foi demonstrado de uma forma esplendorosa!
     Seguindo o método do Igor, solucionarei o problema utilizando o conceito de probabilidade mais banal:
    Vamos calcular a possibilidade de encontrar no máximo 2 bolas pretas:
    P(X≤2) = P(X=0) + P(X=1) + P(X=2) 
    b = brancas
    p = pretas
    a = amarelas
    Assim: Para  P (p=0), temos as seguintes formas:
    (b,b,b) = 3/10 . 3/ 10 . 3/ 10 = 27 / 1000
    (a,a,a) = 3/10 . 3/ 10 . 3/ 10 = 27 / 1000
    (b,b,a) = 3/10 . 3/ 10 . 3/ 10 = 27 / 1000
    (a,b,b) = 3/10 . 3/ 10 . 3/ 10 = 27 / 1000
    (b,a,b) = 3/10 . 3/ 10 . 3/ 10 = 27 / 1000
    (b,a,a) = 3/10 . 3/ 10 . 3/ 10 = 27 / 1000
    (a,a,b) = 3/10 . 3/ 10 . 3/ 10 = 27 / 1000
    (a,b,a) = 3/10 . 3/ 10 . 3/ 10 = 27 / 1000
    P (p=0) = 8*27/1000 = 216/1000
    Para P (p=1), temos as seguintes formas:
    (p,b,b) = 4/10 . 3/ 10 . 3/ 10 = 36 / 1000
    (p,b,a) = 4/10 . 3/ 10 . 3/ 10 = 36 / 1000
    (p,a,b) = 4/10 . 3/ 10 . 3/ 10 = 36 / 1000
    (p,a,a) = 4/10 . 3/ 10 . 3/ 10 = 36 / 1000
    (b,p,a) = 3/10 . 4/ 10 . 3/ 10 = 36 / 1000
    (a,p,b) = 3/10 . 4/ 10 . 3/ 10 = 36 / 1000
    (b,p,b) = 3/10 . 4/ 10 . 3/ 10 = 36 / 1000
    (a,p,a) = 3/10 . 4/ 10 . 3/ 10 = 36 / 1000
    (a,a,p) = 3/10 . 3/ 10 . 4/ 10 = 36 / 1000
    (b,b,p) = 3/10 . 3/ 10 . 4/ 10 = 36 / 1000
    (a,b,p) = 3/10 . 3/ 10 . 4/ 10 = 36 / 1000
    (b,a,p) = 3/10 . 3/ 10 . 4/ 10 = 36 / 1000
    P(p=1) = 12*36 = 432/1000
    Para P(p=2), temos as seguintes formas:
    (p,p,a) = 4/10 . 4/ 10 . 3/ 10 = 48 / 1000
    (p,p,b) = 4/10 . 4/ 10 . 3/ 10 = 48 / 1000
    (p,a,p) = 4/10 . 3/ 10 . 4/ 10 = 48 / 1000
    (p,b,p) = 4/10 . 3/ 10 . 4/ 10 = 48 / 1000
    (b,p,p) = 3/10 . 4/ 10 . 4/ 10 = 48 / 1000
    (a,p,p) = 3/10 . 4/ 10 . 4/ 10 = 48 / 1000
    Para P(p=2) = 6*48 = 288/1000
     Assim: P(p≤2) (216+432+288)/1000 = 0,936
     Provando que o método que o Igor fez está correto...a probabilidade de encontrarmos o valor de P(p≤2) = 1 - P(p=3) = 1 - [4/10 . 4/ 10 . 4/ 10 = 64 / 1000] = 0,936
    Qual método seria o mais adequado para a prova?! Lembrando que existe um fatorzinho nada trivial chamado tempo... rsrs
    Igor óbvio!!
    PORÉM, NÃO ESQUECER QUE NA PROVA QUE PODE SER PEDIDO P(X=1) = ?
    Para X equivalento o parâmetro = Bolas pretas!
    Nessa caso, melhor resolução é do meu xará André!!
    O negócio é não matar a apenas questão... mas, sim o conceito da matéria para não ter que ficar retomando a mesma questão e o mesmo estudo!
    Estamos aqui para aprender realmente estatística!! Na hora da prova, é melhor ir com todas as armas! Ir preparado só com as questões fáceis, manjadas e pelas formuletas não matam a maioria das questão de estatística, principalmente as do Bacen ou SUSEP!!

    Abraços!
  • SO QUERO SABER QUEM É QUE VAI TER TEMPO PRA FAZER UM CÁLCULO DESSE TAMANHO IGUAL DOS DOIS COMENTÁRIOS ACIMA!!! PARABÉNS IGOR PELA SUA RESOLUÇAO!! PROVA TEM QUE SER RÁPIDO PRA GANHA TEMPO..
  • Prezados,

    Não consegui entender a afirmação:

    "Como quero no máximo que 2 sejam pretas eu faço 1 - 0,064 = 0,936"

    Poderiam detalhar esta definição? Desde já agradeço.
  • Thiago,

    A resolução de P(máx duas pretas) = 1 - P(três pretas) se baseia em "eventos complementares"

    Você concorda que a P(nenhuma preta) + P(uma preta) + P(duas pretas) + P(três pretas) = 1 ??
    Não há outro resultado possível, ou seja, ao escolher três bolas com reposição teremos nenhuma, uma, duas ou três bolas pretas. 
    Você concorda que P(máximo duas pretas) = P(nenhuma preta) + P(uma preta) + P(duas pretas) ??
    Compare os dois trechos assinalados com marca-texto.

    Substituindo no primeito trecho assinalado, P(máx duas pretas) + P(três pretas) = 1; e isolando P(máx duas pretas) = 1 - P(três pretas)

    Espero ter ajudado
    Abs
  • É facil e não demora muito!
     
    Legenda: BOLA PRETA =  P e
    BOLA DE OUTRA COR = X.

    Probabilidade de NENHUMA  BOLA  ser preta.

    P (X X X) = 6/10* 6/10 * 6/10
    P (X X X) = 0,216

    Probabilidade de 1  BOLA  ser preta. 

    P (P X X) = 4/10 * 6/10 * 6/10 * 3 ( Lembrar a bola preta pode ser a primeira, a segunda ou a terceira)
    P (P X X) = 0,432

    Probabilidade de 2  BOLAS  serem pretas. 

    P (P P X) = 4/10 * 4/10 * 6/10 * 3 ( Lembrar a bola preta pode ser a primeira e a segunda,  ou a primeira e a terceira, ou a segunda ou a terceira)
    P (P P X) = 0,288

    Agora é só somar: 0,216 + 0,432 + 0,288 = 0,936!
  • Sejam os eventos

    A: No máximo duas bolas são pretas

    (~A) : complementar(negação) de A (as três bolas são prestas):

    P(~A) = 4/10 x 4/10 x 4/10 = 64/1000=0,064  (lembre que o evento é com reposição! o espaço amostral é sempre 10 em cada etapa da retirada)

    P(A) + P(~A) = 1

    P(A) = 1 - P(~A) 

    P(A) = 1 - 0,064 = 0,936

  • gente, PALMAS PARA O IGOR, O CARA TROCOU IDEIA COM PROBABILIDADE DO EVENTO COMPLEMENTAR FACILICITANDO ALGO DE FORMA PRECISA, OU SEJA, FOI EFICIENTE. OBRIGADO, IRMÃO!!!!


ID
642934
Banca
FCC
Órgão
TCE-PR
Ano
2011
Provas
Disciplina
Estatística
Assuntos

Em uma fábrica existem 3 máquinas A, B e C que produzem diariamente 10.000 peças. Sabe-se que A, B e C produzem, respectivamente, 2000, 5000 e 3000 peças. Da produção de A, B e C, respectivamente, 5%,10% e 20% são defeituosas. Seleciona-se uma peça ao acaso e verifica-se que é defeituosa. A probabilidade dela ser proveniente da máquina C é

Alternativas
Comentários
  • A questão é simples, vamos lá:

    Peças Produzidas:                                          

    A: 2000
    B: 5000
    C: 3000


    Da produção sempre saem defeituosas:

    5% de A =  2000*5% = 100
    10% de B = 5000*10% = 500
    20% de C = 3000*20% = 600
    Total de defeituosas = 100 + 500 + 600 = 1200

    Tá no enunciado que a peça é defeituosa, desta forma se é defeituosa é uma das 1200

    Se colocar todas as defeituosas num pote, este terá 1200 peças, sendo que 600 peças serão da C:

    Então: 600/1200 =  0,5 ou 50%

    A probabilidade de ser A é 0,0833
    A probabilidade de ser B é 0,4166

    A LETRA E está CORRETA
  • alguem poderia me ajudar ?
    dois levantamentos independentes mostraram que, de todos os processos abertos em uma vara:
    -35% levaram mais de 18 meses pra serem concluidos 
    45% envolviam questões criminais 
    qual a porcetagem de dprocessos que levaria mais de 18 meses para serem concliudos e também envolviam alguma questão criminal?
  • Utilizando o Teorema de Bayes pois se quer encontrar a probabilidade condicional:
     
    Máquina A - Produz 2.000 peças, ou seja, 20% 
                        Prob. Peças Defeituosas => 0,05 X 0,2
                        Prob. Peças Normais => 0,95 X 0,2

    Máquina B - Produz 5.000 peças, 50%
                        Prob. Peças Defeituosas => 0,10 X 0,5
                        Prob. Peças Normais => 0,90 X 0,5

    Máquina C - Produz 3.000 peças, 30%
                         Prob. Peças Defeituosas => 0,2 X 0,3
                         Prob. Peças Normais => 0,80 X 0,3
     
    P(C|Defeituosa) => Se pede a probabilidade da peça ser produzida pela máquina C sabendo que é defeituosa

                                                     0,2 X 0,3                                              0,06
     P(C|Defeituosa) =     ______________________________  =      ________ = 0,50
                                        
                                         0,05 X 0,2 + 0,10 X 0,5 + 0,20 X 0,3              0,12












     

  • MAQUINA A = 2000p (5%) = 100 defeituosas

    MAQUINA B = 5000p (10%) = 500 defeituosas

    MAQUINA C = 3000P (20%) = 600 defeituosas

    TOTAL PEÇAS DEFEITUOSAS = 1200

    Probabilidade = parte interesse/total

    P = 600/1200 = 0,50


ID
670777
Banca
CONSULPLAN
Órgão
TSE
Ano
2012
Provas
Disciplina
Estatística
Assuntos

Sobre os modelos de análise de dados discretos é correto afirmar que

Alternativas
Comentários
  • modelo probit tá relacionado com a normal padronizada:

    http://www.fep.up.pt/disciplinas/2E103/modelos_de_escolha_binaria.pdf


ID
670795
Banca
CONSULPLAN
Órgão
TSE
Ano
2012
Provas
Disciplina
Estatística
Assuntos

Em um grupo de 20 bolas, 5 são vermelhas e 15 verdes. A probabilidade, com aproximação de duas casas decimais, de que sejam retiradas três bolas vermelhas em sequência, sem reposição, é

Alternativas
Comentários
  • Pr (3BV) = 5/ 20 * 4/ 19 * 3/ 18 = 1/ 114 = 0,0087719 = 0,88%

ID
670900
Banca
CONSULPLAN
Órgão
TSE
Ano
2012
Provas
Disciplina
Estatística
Assuntos

A equipe de pesquisa de um laboratório farmacêutico está desenvolvendo um medicamento analgésico que promete aliviar a dor de cabeça em um tempo médio menor do que o tempo gasto pelo medicamento padrão, que é de 15 minutos, em média. Para liberar o novo medicamento com essa promessa, é necessário executar um experimento e analisar os dados coletados. Depois de planejar e executar o experimento com a nova droga, coletar os dados e processá-los, o teste estatístico apropriado, que adotou uma hipótese alternativa unilateral, resultou em um valor-p (ou probabilidade de significância) igual a 0,028. Na definição das hipóteses do teste, levou-se em conta que o erro de liberar o medicamento com uma falsa promessa de redução no tempo de alívio da dor de cabeça é mais grave do que deixar de liberar um novo medicamento que funcione em um tempo menor. Denotando por µ o tempo médio, em minutos, para o alívio da dor de cabeça do novo medicamento, considere que

I. as hipóteses nula e alternativa do teste estatístico são, respectivamente, (µ  ≥ 15) e (µ < 15).

II. adotando-se um nível de significância de 0,05, há evidências estatísticas suficientes contra a hipótese nula do teste.

III. se a hipótese alternativa do teste fosse bilateral, o valor- p seria igual a 0,014.

Assinale

Alternativas
Comentários
  • Se não foi fornecido o Desvio-Padrão, como se pode afirmar que a letra B está correta?

    Há como se calcular o DP com os dados do problema?

  • I - CORRETO. Pois a hipótese nula é a formulada pela equipe é o que queremos e a hipótese alternativa é o que vai de encontro com essa ideia, ou seja, da história contada percebemos que estamos investigando o caso mais grave é onde estamos interessado, por isso H0 é igual ou maior que 15.

    II - CORRETO. Se o (valor-p > alpha) eu rejeito a hipótese nula. A ideia é o seguinte, alpha é o seu nível de significância do teste (quem escolhe é o pesquisador) você pode ter até esse valor do valor-p para continuar aceitando H0, isto quer dizer que neste caso se eu tivesse um alpha de 2,8% e um valor-p de 2,8% eu aceitaria H0.

    III - ERRADO. Pois eu deveria multiplicar por dois o p-value unicaudal e o enunciado está dividindo.

    RESPOSTA B

ID
694006
Banca
FCC
Órgão
Prefeitura de São Paulo - SP
Ano
2012
Provas
Disciplina
Estatística
Assuntos

Suponha que ao realizar um experimento ocorra o evento A com probabilidade p e não ocorra com probabilidade (1-p). Repetimos o experimento de forma independente até que A ocorra pela primeira vez. Seja: X = número de repetições do experimento até que A ocorra pela primeira vez. Sabendo que a média de X é 3, a probabilidade condicional expressa por P (X = 2 |X ≤ 3) é igual a

Alternativas
Comentários
  • Gabarito Correto: Letra C
      Distribuição Geométrica é caracterizada por essa característica: em n tentativas, qual é a probabilidade de que o que vc quer somente aconteça na enésima vez.  Para isso, ela tem que falhar nas (n-1) tentativas anteriores.  Se a probabilidade de ocorrer o que vc quer é p, então a probabilidade de não acontecer o que vc quer é (1 - p).  

    Logo a fórmula é :  P = (1 - p)n-1 . p

      A média é dada E(x) = 1/p e a variância é dada por  Var(x) = (1-p)/p2

      Como a banca disse que a média é 3 então:  E(x) = 1/p

       3 = 1/p

       Logo p = 1/3 , portanto 1 - p = 1 - 1/3 = 2/3.

    A probabilidade condicional afirma que P (x < = 3) = P(X=1) + P(X=2) + P(X=3)

    P(X=1) = o fato aconteceu na primeira tentativa = 1/3
    P(X=2) = o fato aconteceu na segunda tentativa, logo a primeira falhou e a segunda foi bem sucedida = 2/3 x 1/3 = 2/9
    P(X=3) = o fato aconteceu na terceira tentativa, logo as duas primeiras falharam e a terceira foi bem sucedida = 2/3 x 2/3 x 1/3 = 4/27
    Logo  P( X = 2 | X < = 3) = (2/9)/(19/27) = (2/9) x (27/19) = 6/19

  • Olá,

    o ENUNCIADO da questão aqui no site está ERRADO!

    A probabilidade condicional que a questão exige é "P ( x = 2 | x <= 3 )" e não "P ( x = 2 | x = 3)".

    Pelo menos é como está aparecendo para mim.

    Apenas para esclarecer mais o final da explicação do colega anterior...

    Trata-se de uma distribuição Geométrica - como ele bem explicou - e iremos usá-la para calcular as probabilidades do modelo abaixo:

    P ( X=2 | X<=3 ) = P[ (X=2) ∩ (X<=3)] / P(X<=3)

    No caso dessa questão, a intercessão "P[ (X=2) ∩ (X<=3)]" = P( X = 2 ). Daí:

    P ( X=2 | X<=3 ) = P(X=2) / P(X<=3)

    Aí é só aplicar a distribuição...

    Bons estudos a todos!

  • Excelentes explicações!

  • https://exatasparaconcursos.wordpress.com/2012/09/21/iss-sp-2012-distribuio-geomtrica/


ID
698338
Banca
FCC
Órgão
TRE-SP
Ano
2012
Provas
Disciplina
Estatística
Assuntos

Sabe-se que A, B e C são eventos independentes, associados a um mesmo espaço amostral, com probabilidades dadas, respectivamente, por 1⁄3 e 1⁄5, 1⁄2 . A probabilidade de que exatamente dois desses eventos ocorram é igual a

Alternativas
Comentários
  • = 1/3.1/5 + 1/3.1/2 + 1/5.1/2 - 3(1/3.1/5.1/2)
    = 1/15 + 1/6 + 1/10 - 3/30
    = 14/60
    = 7/30

    RESPOSTA C

    PS: Para visualizar melhor sugiro que se faça o desenho do diagrama de Venn.
  • Probabilidade de A ocorrer = 1/3 e de A não ocorrer = 2/3
    Probabilidade de B ocorrer = 1/5 e de B não ocorrer = 4/5
    Probabilidade de C ocorrer = 1/2 e de C não ocorrer = 1/2
    Então a probabilidade de que dois eventos ocorram é que:
      1)   A e B ocorram e C não ocorra = 1/3 * 1/5 * 1/2 = 1/30
    OU (+)
      2)   A e C ocorram e B não ocorra = 1/3 * 1/2 * 4/5 = 4/30
    OU (+)
      3)   B e C ocorram e A não ocorra = 1/5 * 1/2 * 2/3 = 2/30
    Logo   1   +  2  + 3 =   1/30 + 4/30 + 2/30 = 7/30
  • O Diagrama de Venn oferece uma vizualização melhor para a solução dessa questão:

    https://www.google.com.br/search?q=diagrama+de+venn&hl=pt-BR&prmd=imvnsb&tbm=isch&tbo=u&source=univ&sa=X&ei=j2xtUIWKOImW8gS2moCIDg&ved=0CCQQsAQ&biw=784&bih=485

  • Lembrando sempre que o "e" tem valor de multiplicação e o "ou" de soma.


    A questão quer saber a probabilidade de exatamente dois eventos, entre A,B, e C, ocorrerem.


    Então tem que ocorrer

    A e B

    OU

    A e C

    OU

    B e C


    Essas 3 são todas as possibilidades diferentes de dois eventos ocorrerem nesse conjunto.


    Dessa forma, fazemos:

    A e B =

    Ocorre A e Ocorre B e Não Ocorre C =

    1/3 * 1/5 * 1/2 = 1/30


    A e C =

    Ocorre A e Ocorre C e Não Ocorre B =

    1/3 * 1/2 * 4/5 = 4/30


    B e C =

    Ocorre B e Ocorre C e Não Ocorre A =

    1/5 * 1/2 * 2/3 = 2/30


    Agora basta somar: 1/30 + 4/30 + 2/30 = 7/30


    Bons estudos!

  • é fácil multiplica os denominadores e soma cruzando os dois primeiro e subtrai com ultimo.


ID
698341
Banca
FCC
Órgão
TRE-SP
Ano
2012
Provas
Disciplina
Estatística
Assuntos

Sabe-se que 80% de todos os eleitores de uma grande cidade brasileira são favoráveis que se aplique, nas próximas eleições, a Lei da Ficha Limpa. Se 4 eleitores são selecionados ao acaso e com reposição dentre todos os eleitores dessa cidade, a probabilidade de que pelo menos 3 sejam favoráveis que a referida lei seja aplicada nas próximas eleições é

Alternativas
Comentários
  • FL = 80%=0,8   ÑF = 20%= 0,2

    0,8*0,8*0,8*0,8*0,2=0,8192
  • 4(0,8*0,8*0,8*0,2) - são quatro combinações diferentes para que ocorra: três favoraveis e um não favoravel

    (0,8*0,8*0,8*0,8) - apenas uma combinação para que ocorra: todos favoraveis.

    4(0,8*0,8*0,8*0,2) + (0,8*0,8*0,8*0,8)= 0,4096 + 0,4096 = 0,8192


     

  • Bom,depois de um tempo pensando eu somente consegui chegar a essa conclusão:


    são 4 pessoas e a questão pede que no minimo 3 sejam favoráveis a ficha limpa,então entende-se que pode tanto ser as 4  pessoas favoráveis ou apenas 3.Vamos então supor que o total de pessoas seja 10  e 80% então corresponda a 8 pessoas.Então favoraveis são 8/10 e contra são 2/10
    Como o exercicio fala em reposição o total não ira mudar.
    Temos 4 posições a ser preenchidas.
     
    1ºPOSIÇÃO : 8/10
    2º POSIÇÃO:8/10
    3ºPOSIÇÃO:8/10
    4ºPOSIÇÃO:8/10 ou 2/10

    8/10 x 8/10 x 810 x (8/10x2/10) = 0,08192 Bom, na resposta correta da questão o valor que  aparece é  0,8192... diferente do  meu resultado .não sei se esse raciocinio está correto mas foi o mais proximo que consegui chegar.


  • Apenas para complementar os comentários dos colegas acima:
    quando a questão fala em "pelo menos 3" significa no mínimo 3, ou seja, pode ser 3 favoráveis ou 4 favoráveis.
    Considere F para favorável e N para não favorável
    1 opção = F x F x F x N => 0,8 x 0,8 x 0,8 x 0,2 x 4!/3! => já que são 4 opções com repetição de 3F -  a ordem altera
    1 opção = 1024/10000 x 4 = 4096/10000
    2opção = F x F x F x F = 0,8 x 0,8 x 0,8 x 0,8 = 4096/10000 => aqui a ordem não importa já que são iguais

    opção 1 + opção 2 = 0,8192

    Gab. A

    Bons estudos !!
  • Alguém pode ajudar a recordar porque não é possível aplicar o método de distribuição binomial nesta questão?! Obrigado!!!
  • Respondendo ao julio ... é possível sim

    p (sucesso) = 0,8
    q (fracasso) = 1 - 0,8 = 0,2
    k = 3 e 4
    n = 4

    Três sucessos: P(x=3) = C(3,4).p^3.q^1 = 0,4096
    Quatro sucessos: P(x=4) = C(4,4).p^4.q^0 = 0,4092

    Somando: 0,4096 + 0,4096 = 0,8192
  • Probabilidades (S: Ser a favor / N: Não ser a favor)

    S ; S ; S ; S = 0,8 * 0,8 * 0,8 * 0,8 = 0,4096

    S ; S ; S ; N = 0,8 * 0,8 * 0,8 * 0,2 = 0,1024

    S ; S ; N ; S = 0,8 * 0,8 * 0,2 * 0,8 = 0,1024

    S ; N ; S ; S = 0,8 * 0,2 * 0,8 * 0,8 = 0,1024

    N ; S ; S ; S = 0,2 * 0,8 * 0,8 * 0,8 = 0,1024


    Somando todas as probabilidades, P = 0,8192! Lembrando que se são pelo menos 3, deve-se considerar a possibilidade dos 4 escolhidos serem a favor.

  • Como que eu sei que a ordem importa nesse caso ? Eu fiz 0,8. 0,8.0,8.0,8 + 0,8.0,8.0,8.0,2   Esqueci de multiplicar por 4 essa última operação.Eu sempre confundo quando que a ordem conta, quando não conta.


ID
698344
Banca
FCC
Órgão
TRE-SP
Ano
2012
Provas
Disciplina
Estatística
Assuntos

Numa determinada zona eleitoral sabe-se que 40% dos eleitores são do sexo masculino. Entre estes, 10% têm curso superior ao passo que entre os eleitores do sexo feminino, 25% têm curso superior. Selecionando-se um eleitor ao acaso, a probabilidade de que ele seja do sexo feminino ou não tenha curso superior é

Alternativas
Comentários
  • Pr(H) = 40% = 0,4
    Pr(M) = 60% = 0,6
    Pr(H e S) = 10% de 40% = 0,1 * 0,4 = 0,04
    Pr(H e NS) = 90% de 40% = 0,9 * 0,4 = 0,36
    Pr(M e S) = 25% de 60% = 0,25 * 0,6 = 0,15
    Pr(M e NS) = 75% de 60% = 0,75 * 0,6 = 0,45
    Pr (NS) = 45% + 36% = 0,45 + 0,36 = 0,81
    Pr (M ou NS) = Pr(M) + Pr(NS) - Pr(M e NS) = 0,6 + 0,81 - 0,15 = 0,96
  • Adicionando somente um comentario sobre o comentario anterior:
    Como a questão quer saber a probabilidade de ser feminino ou não tenha curso superior seria o mesmo que afirmar que poderia ser feminino superior mais todos os que não tenham curso superior.
    Assim ficaria
    P(F/ns)+P(M/ns)+P(F/s) = 0,45 + 0,36 + 0,15 = 0,96 ou 96%
  • É mais objetivo pegar o complementar:

    Evento que não tenho interesse: ser homem e ter nível superior = 0,4*0,1 = 0,04

    Resposta: 1 - 0,04 = 0,96 >> letra D


ID
698347
Banca
FCC
Órgão
TRE-SP
Ano
2012
Provas
Disciplina
Estatística
Assuntos

O custo para a realização de um experimento é de 500 reais. Se o experimento falhar haverá um custo adicional de 100 reais para a realização de uma nova tentativa. Sabendo-se que a probabilidade de sucesso em qualquer tentativa é 0,4 e que todas são independentes, o custo esperado de todo o procedimento até que o primeiro sucesso seja alcançado é

Alternativas
Comentários
  • X = nº de tentativas até o 1º sucesso (Distribuição Geométrica)
    P(X=1)  -> Custo esperado: R$500
    P(X=2) -> Custo esperado: R$500+R$600
    P(x=3) -> Custo esperado: R$500+R$600+600
    .
    .
    .
    P(X=n) -> Custo esperado: R$500+600(n-1)

    E(X) = 1/p (Média da distribuição geométrica)
    E(X)=1/0,4=2,5. Logo Custo esperado = 500+600(2,5-1)= 1400
  • E(X) = 1/p (Média da distribuição geométrica)
    E(X)=1/0,4=2,5

    custo esperado = 2,5*500 + (2,5 -1)*100 = 1400
    obs: (2,5 - 1), pois, na primeira tentativa não há custo adicional de 100 reais. Daí subtrair 1 ensaio

  • cara.... achei mal redigida a questão...

    entendi que seriam gastos 500 reais na primeira e depois 100 reais adicionais cada nova tentativa que fosse feita.

    ou seja 

    na primeira 500

    na segunda 100

    na terceira 100  e etc.

     

    além de que, considerar o 0,5 como metade do valor gasto não faz muito sentido já que não existe "meia tentativa" 

    ou ele faz outra tentativa e gasta mais 600 ou ele não faz outra tentativa e não gasta nada, não faz sentido o 0,5 como 300 reais

  • Pensei exatamente como você @bruno ali


ID
698380
Banca
FCC
Órgão
TRE-SP
Ano
2012
Provas
Disciplina
Estatística
Assuntos

A função de probabilidade conjunta das variáveis X e Y é dada por: f (x,y) = 1&frasl;32 (x2 + y2 ), x = 0,1,2,3 e y = 0,1 Nessas condições, a média de Y e P(X + Y = 3) são dados, respectivamente, por


Alternativas
Comentários
  • Antes de encontrar a média de y, é necessário encontrar f(y), a qual é dada por:

    integral f(x,y) dx

    Depois E(y) = integral de y* f(y) dy

    P(x + y = 3) >> basta substituirmos  (3,0) e (2,1) em f(x,y) e somar os resultados dessas substituicoes, ou seja:

    f(3,0) = 9/32

    f(2,1) = 5/32

    sendo assim, P(x + y = 3) = 9/32 + 5/32 = 7/16


     


ID
722638
Banca
FCC
Órgão
TRT - 6ª Região (PE)
Ano
2012
Provas
Disciplina
Estatística
Assuntos

A caixa A tem 5 cartas numeradas de 1 a 5. A caixa B tem 8 cartas numeradas de 1 a 8. A caixa C tem 10 cartas numeradas de 1 a 10. Uma caixa é selecionada ao acaso e uma carta é retirada. Se o número da carta é impar, a probabilidade de a carta selecionada ter vindo da caixa B é

Alternativas
Comentários
  • A: 1,2,3,4,5,

    probabilidade de ter impar em A = 3/5,

    B: 1,2,3,4,5,6,7,8,

    probabilidade de ter impar em B = 4/8,

    C: 1,2,3,4,5,6,7,8,9,10,

    probabilidade de ter impar em C = 5/10,

    probabilidade de ter vindo impar de B sabendo que é impar:

    (4/8) / (3/5 + 4/8 + 5/10) = 5 /16




ID
722644
Banca
FCC
Órgão
TRT - 6ª Região (PE)
Ano
2012
Provas
Disciplina
Estatística
Assuntos

As probabilidades de um contador, A, demorar uma, duas ou três horas para preencher uma declaração de imposto de renda são dadas, respectivamente, por 1/4, 1/2  e 1/4 . Dentre 5 declarações escolhidas aleatoriamente e com reposição, das declarações que A deverá elaborar, a probabilidade dele demorar para o preenchimento, em três delas 1 hora, em uma 2 horas e na restante 3 horas, é igual a

Alternativas
Comentários
  • pense em 5 caixinhas

    a primeira observacao (3 horas) pode ocupar 5 posicoes

    a segunda observacao (2 horas) 4 posicoes

    e o conjunto formado por 3 observações (de 1 hora), restará ocupar somente uma posição. pois as demais observações já foram variadas

    sendo assim a resposta será:


    5*4*1*(1/4 ^ 3) * 1/2 ^ 1 * 1/4 ^ 1 = 5/128



ID
722647
Banca
FCC
Órgão
TRT - 6ª Região (PE)
Ano
2012
Provas
Disciplina
Estatística
Assuntos

A probabilidade de que no quinto lançamento de um dado não viciado (numerado de 1 a 6) ocorra a face 3 pela segunda vez é

Alternativas
Comentários
  • (5 2)*(1/6)^2 *(5/6)^3 = 125 / 1944

     

  • [combinação 4:1] *(1/6)^2*(5/6)^3=125/1944, cmbinação 4:1 visto que o quinto lançamento sabemos que é 3, portanto nos resta apenas as combinações anteriores.